[obm-l] Re: [obm-l] Re: [obm-l] Pesagens ( Balança Eletrônica)

2023-11-19 Por tôpico Jeferson Almir
Eu tinha errado umas contas, mas sua cota está correta Ralph, preciso
montar um exemplo com 21 pesagens

Em dom., 19 de nov. de 2023 às 15:00, Jeferson Almir <
jefersonram...@gmail.com> escreveu:

> Pelo visto, está sim Ralph!! Agora temos que montar uma estratégia que com
> 21 pesagens.
>
> Em dom., 19 de nov. de 2023 às 13:55, Ralph Costa Teixeira <
> ralp...@gmail.com> escreveu:
>
>> Existem 2022*2021/2 possibilidades para as 2 falsas. Qualquer estratégia
>> que seja criada com k pesagens que dão apenas 2 respostas cada distingue no
>> máximo dentre 2^k possibilidades. Então devemos ter 2^k >= 2022*2021/2...
>> hmm, isso daria k como pelo menos 21? Errei algo?
>>
>> On Sun, Nov 19, 2023, 12:16 Jeferson Almir 
>> wrote:
>>
>>> Amigos, encontrei como K mínimo o valor 11 mas desconfio que seja menos.
>>> Se alguém souber uma ideia que acabe o problema serei grato.
>>>
>>>
>>> Em Villa Par todas as moedas autênticas pesam uma quantidade par de
>>> gramas e todas as moedas falsas pesam uma quantidade impar de gramas.
>>>
>>> Se você tiver 2022 moedas entre as quais sabe que exatamente 2 são
>>> falsas.
>>>
>>> Se tiver uma balança eletrônica que informe apenas se o peso total dos
>>> objetos colocados nela é par ou impar.
>>>
>>> Determine o valor mínimo de k para qualquer estratégia que permita
>>> identificar as moedas falsas usando a balança  no máximo k vezes.
>>>
>>> --
>>> Esta mensagem foi verificada pelo sistema de antivírus e
>>> acredita-se estar livre de perigo.
>>
>>
>> --
>> Esta mensagem foi verificada pelo sistema de antivírus e
>> acredita-se estar livre de perigo.
>
>

-- 
Esta mensagem foi verificada pelo sistema de antiv�rus e
 acredita-se estar livre de perigo.



[obm-l] Re: [obm-l] Re: [obm-l] Pesagens ( Balança Eletrônica)

2023-11-19 Por tôpico Jeferson Almir
Pelo visto, está sim Ralph!! Agora temos que montar uma estratégia que com
21 pesagens.

Em dom., 19 de nov. de 2023 às 13:55, Ralph Costa Teixeira <
ralp...@gmail.com> escreveu:

> Existem 2022*2021/2 possibilidades para as 2 falsas. Qualquer estratégia
> que seja criada com k pesagens que dão apenas 2 respostas cada distingue no
> máximo dentre 2^k possibilidades. Então devemos ter 2^k >= 2022*2021/2...
> hmm, isso daria k como pelo menos 21? Errei algo?
>
> On Sun, Nov 19, 2023, 12:16 Jeferson Almir 
> wrote:
>
>> Amigos, encontrei como K mínimo o valor 11 mas desconfio que seja menos.
>> Se alguém souber uma ideia que acabe o problema serei grato.
>>
>>
>> Em Villa Par todas as moedas autênticas pesam uma quantidade par de
>> gramas e todas as moedas falsas pesam uma quantidade impar de gramas.
>>
>> Se você tiver 2022 moedas entre as quais sabe que exatamente 2 são falsas.
>>
>> Se tiver uma balança eletrônica que informe apenas se o peso total dos
>> objetos colocados nela é par ou impar.
>>
>> Determine o valor mínimo de k para qualquer estratégia que permita
>> identificar as moedas falsas usando a balança  no máximo k vezes.
>>
>> --
>> Esta mensagem foi verificada pelo sistema de antivírus e
>> acredita-se estar livre de perigo.
>
>
> --
> Esta mensagem foi verificada pelo sistema de antivírus e
> acredita-se estar livre de perigo.

-- 
Esta mensagem foi verificada pelo sistema de antiv�rus e
 acredita-se estar livre de perigo.



[obm-l] Pesagens ( Balança Eletrônica)

2023-11-19 Por tôpico Jeferson Almir
Amigos, encontrei como K mínimo o valor 11 mas desconfio que seja menos. Se
alguém souber uma ideia que acabe o problema serei grato.


Em Villa Par todas as moedas autênticas pesam uma quantidade par de gramas
e todas as moedas falsas pesam uma quantidade impar de gramas.

Se você tiver 2022 moedas entre as quais sabe que exatamente 2 são falsas.

Se tiver uma balança eletrônica que informe apenas se o peso total dos
objetos colocados nela é par ou impar.

Determine o valor mínimo de k para qualquer estratégia que permita
identificar as moedas falsas usando a balança  no máximo k vezes.

-- 
Esta mensagem foi verificada pelo sistema de antiv�rus e
 acredita-se estar livre de perigo.



[obm-l] Perguntas mínimas no Tabuleiro

2022-09-13 Por tôpico Jeferson Almir
Os números de 1 a 49 são arbitrariamente dispostos num tabuleiro quadrado
7x7 . Podemos escolher qualquer quadrado composto de múltiplas células e
perguntar quais números estão contidos nele. Ao menos quantas perguntas são
necessárias para determinarmos a configuração exata dos números?

Alguém tem uma ideia ótima ??

-- 
Esta mensagem foi verificada pelo sistema de antiv�rus e
 acredita-se estar livre de perigo.



[obm-l] Números de tentativas

2021-12-13 Por tôpico Jeferson Almir
Amigos peço ajuda nessa questão.

Tem uma senha de 3 digitos
(Qualquer digito  de 0 a 9)
E nos temos um dispositivo
Que compara a senha
Com um número que escolhemos
E retorna não se tem todos os digitos diferentes da senha
E retorna quase se tem pelo menos 1 digito coincidente com a senha
Qual é o menor numero de tentativas que precisamos usar esse dispositivo
tal que podemos descobrir a senha com certeza, independente de qual ela
seja?

-- 
Esta mensagem foi verificada pelo sistema de antiv�rus e
 acredita-se estar livre de perigo.



[obm-l] Sequência Injetiva

2021-02-13 Por tôpico Jeferson Almir
Amigos, peço ajuda em provar a injetividade dessa sequência que seria uma
saída para provar a unica ocorrência do racional que aparece nela. Estou
andando em círculos tentando montar uma possível indução.


Dado a sequência a_1 = 1 e a_2n = a_n  + 1 e a_2n+1 = 1/a_2n.

Prove que para todo racional positivo que ocorre na sequência, ocorre uma
única vez.

-- 
Esta mensagem foi verificada pelo sistema de antiv�rus e
 acredita-se estar livre de perigo.



[obm-l] Usamo ( polinômios )

2020-09-10 Por tôpico Jeferson Almir
(USAMO) Prove que qualquer polinômio mônico de grau n, com coeficientes
reais, pode ser escrito como média aritmética de dois polinômios mônicos de
grau n com n raízes reais cada.

O material sugere usar o polinômio interpolador de Lagrange.
Alguém teria uma solução pra isso ?? via polinômio de Lagrange.?

-- 
Esta mensagem foi verificada pelo sistema de antiv�rus e
 acredita-se estar livre de perigo.



[obm-l] Polinômios - Longlists -83

2020-06-21 Por tôpico Jeferson Almir
Amigos peço ajuda no seguinte problema( item b principalmente).

Considere a expansão
( 1 + x + x^2 + x^3 + x^4 )^496 = a_0 + a_1x +  + a_1984x^1984

a) Determine o mdc( a_3, a_8, a_13, ... , a_1983 )

b) Prove que 10^340 < a_922 < 10^347

No item a) eu usei raizes da unidade, mas se alguém tem alguma ideia via
Funções Geratrizes eu agradeceria muito. Já o item b) tentei usar médias ou
algo de termo geral de expansão multinomial e não consegui nada.

-- 
Esta mensagem foi verificada pelo sistema de antiv�rus e
 acredita-se estar livre de perigo.



[obm-l] Re: [obm-l] Re: [obm-l] Encontrar K mínimo

2020-05-23 Por tôpico Jeferson Almir
Voos finitos = é sempre possível chegar com uma certa quantidade de voos.
 Os casos iniciais que fiz me pareceu uma conjectura muito “ óbvia “, mas
não tenho certeza.
* não existe Voo  de B para B

Em sáb, 23 de mai de 2020 às 12:54, Bernardo Freitas Paulo da Costa <
bernardo...@gmail.com> escreveu:

> On Sat, May 23, 2020 at 11:46 AM Jeferson Almir
>  wrote:
> >
> > Amigos peço ajuda nesse problema, ou até algum resultado de grafos que
> resolva.
> >
> > Terra Brasilis  possui 2021 cidades, e existem voos de ida e volta entre
> algumas dessas  cidades de maneira que é possível chegar a qualquer outra
> através de voos finitos. Encontre o menor inteiro positivo k tal que,
> independente da configuração dos voos, é possível escolher k cidades de
> modo que qualquer uma das 2021 cidades possui voo direto para alguma das
> cidades marcadas.
>
> Não entendi o que quer dizer "vôos finitos"...  Tem um enunciado mais
> preciso, ou é isso aí?
>
> E, para tentar ajudar: o que acontece se forem 2 cidades? 3?  Aliás,
> uma pergunta, se o caso de 2 cidades for A - B, não tem "vôos de B
> para B", então você teria que escolher as cidades (A,B) ?  Ou também
> está faltando esse detalhe no enunciado?
>
>
> Abraços,
> --
> Bernardo Freitas Paulo da Costa
>
> --
> Esta mensagem foi verificada pelo sistema de antivírus e
>  acredita-se estar livre de perigo.
>
>
> =
> Instru�ões para entrar na lista, sair da lista e usar a lista em
> http://www.mat.puc-rio.br/~obmlistas/obm-l.html
> =
>

-- 
Esta mensagem foi verificada pelo sistema de antiv�rus e
 acredita-se estar livre de perigo.



[obm-l] Encontrar K mínimo

2020-05-23 Por tôpico Jeferson Almir
Amigos peço ajuda nesse problema, ou até algum resultado de grafos que
resolva.

Terra Brasilis  possui 2021 cidades, e existem voos de ida e volta entre
algumas dessas  cidades de maneira que é possível chegar a qualquer outra
através de voos finitos. Encontre o menor inteiro positivo k tal que,
independente da configuração dos voos, é possível escolher k cidades de
modo que qualquer uma das 2021 cidades possui voo direto para alguma das
cidades marcadas.

-- 
Esta mensagem foi verificada pelo sistema de antiv�rus e
 acredita-se estar livre de perigo.



[obm-l] Teoria dos números

2020-04-23 Por tôpico Jeferson Almir
Amigos, peço ajuda nessa questão.

 Sejam a e b inteiros positivos >=2 tal que (a^n)-1|(b^n)-1 pra todos os
inteiros positivos n,mostrar que b é potencia inteira de a.

-- 
Esta mensagem foi verificada pelo sistema de antiv�rus e
 acredita-se estar livre de perigo.



[obm-l] Sequência de inteiros positivos

2019-09-24 Por tôpico Jeferson Almir
Peço ajuda no seguinte problema


Uma sequência infinita x_1 , x_2 , ... x_n  de números inteiros positivos
satisfaz x_ {n + 2} =  M.D.C (x_ {n + 1}, x_n) + 2006  para cada número
inteiro positivo n. Existe uma sequência que contém exatamente 10^{2006}
números distintos?

-- 
Esta mensagem foi verificada pelo sistema de antiv�rus e
 acredita-se estar livre de perigo.



Re: [obm-l] Triplas pitagoricas

2019-08-25 Por tôpico Jeferson Almir
O problema melhor formulado é:

“ prove que não existem inteiros positivos x,y,z,w tais que x^2 + y^2 = z^2
e x^2 - y^2 = w^2 “

Em dom, 25 de ago de 2019 às 11:23, Alexandre Antunes <
prof.alexandreantu...@gmail.com> escreveu:

> Bom dia,
>
> Existe um caso "trivial", com infinitas possibilidades: Sejam a,b números
> do conjunto N (natural)
>
> Se b = 0
>
> a^2 + b^2 = a^2
> a^2  - b^2 = a^2
>
> Em Ter, 13 de ago de 2019 19:29, Jeferson Almir 
> escreveu:
>
>> Como eu provo que não existem 2 naturais cuja soma e diferença de seus
>> quadrados sejam quadrados ?
>>
>> Ps: eu tentei pegar a solução clássica da equação da soma x^2 + y^2 = z^2
>> e tentei jogar na diferença pra aparecer algum absurdo em algum módulo mas
>> obtive sucesso.
>>
>> --
>> Esta mensagem foi verificada pelo sistema de antivírus e
>> acredita-se estar livre de perigo.
>
>
> --
> Esta mensagem foi verificada pelo sistema de antivírus e
> acredita-se estar livre de perigo.

-- 
Esta mensagem foi verificada pelo sistema de antiv�rus e
 acredita-se estar livre de perigo.



Re: [obm-l] Triplas pitagoricas

2019-08-20 Por tôpico Jeferson Almir
Arthur nessa passagem tem um erro de sinal =>
 “ portanto d=g e e^2+f^2=u^2+v^2 “
E na verdade é
 e^2+f^2=u^2- v^2 quando vc iguala o x de um com o x da outra equação. Não
implicando z = x .

Em ter, 13 de ago de 2019 às 22:00, arthurquimu 
escreveu:

> Queremos provar que não existem soluções inteiras não nulas para:
>
> x^2+y^2=z^2 (1)
> x^2-y^2=w^2 (2)
>
> Rearranjando (2), teremos x^2=y^2+w^2
>
> Só que é um fato matemático conhecido de que ternas pitagóricas possuem a
> forma
> x = (u^2 - v^2)d
> y = 2uvd
> z = (u^2+v^2)d
> Para inteiros u, v e d com mdc(u, v)=1. (Fica como exercício para o leitor
> provar que y é par)
>
> Fazendo o mesmo em (2), teremos:
>
> y= 2efg
> w = (e^2-f^2)g
> x = (e^2+f^2)g
>
> Daí vem que 2efg = 2uvd implicando que g=(uvd)/(ef)
>
> Vem também que (e^2+f^2)g = (u^2+v^2)d e, assim, (e^2+f^2)uv = ef(u^2+v^2)
>
> Só que daí
> uv | ef(u^2+v^2)
> uv | ef(u^2+v^2) + ef * 2uv
> uv | ef (u+v)^2
> u | uv | ef(u+v)^2
> u | efv^2
> u | ef
> Analogamente v | ef, só que como mdc(u, v) = 1, então uv | ef
> Analogamente ef | uv, logo ef=uv, portanto d=g e e^2+f^2=u^2+v^2 de sorte
> que z = (u^2+v^2)d = (e^2+f^2)g = x;
> Só que de z=x, vem da equação (1) que y=0, absurdo.
>
>
>
> Enviado do meu smartphone Samsung Galaxy.
>
>  Mensagem original 
> De : Jeferson Almir 
> Data: 13/08/2019 20:06 (GMT-03:00)
> Para: obm-l@mat.puc-rio.br
> Assunto: Re: [obm-l] Triplas pitagoricas
>
> Tem que ser algo do tipo Israel
> x^2 + y^2 = A^2
> x^2 - y^2 = B^2
>
> Em ter, 13 de ago de 2019 às 19:56, Israel Meireles Chrisostomo <
> israelmchrisost...@gmail.com> escreveu:
>
>> opa me desculpe ju errei aqui desculpe -me
>>
>>
>> <http://www.avg.com/email-signature?utm_medium=email_source=link_campaign=sig-email_content=webmail>
>>  Livre
>> de vírus. www.avg.com
>> <http://www.avg.com/email-signature?utm_medium=email_source=link_campaign=sig-email_content=webmail>.
>>
>> <#m_5837217687322342588_m_-7862083770974194334_DAB4FAD8-2DD7-40BB-A1B8-4E2AA1F9FDF2>
>>
>> Em ter, 13 de ago de 2019 às 19:46, Israel Meireles Chrisostomo <
>> israelmchrisost...@gmail.com> escreveu:
>>
>>> Suponha sem perda de generalidade que x,y,z são positivos. Vc tem x^2 +
>>> y^2 = z^2 e x^2 - y^2 = z^2 somando as duas equações temos x^2=z^2 e então
>>> x=z por outro lado subtraindo as duas igualdades y^2=z^2 o que implica que
>>> y=z isso implica que 2x^2=x^2 e então 2=1 o que é um absurdo
>>>
>>>
>>> <http://www.avg.com/email-signature?utm_medium=email_source=link_campaign=sig-email_content=webmail>
>>>  Livre
>>> de vírus. www.avg.com
>>> <http://www.avg.com/email-signature?utm_medium=email_source=link_campaign=sig-email_content=webmail>.
>>>
>>> <#m_5837217687322342588_m_-7862083770974194334_m_-315779286925050189_DAB4FAD8-2DD7-40BB-A1B8-4E2AA1F9FDF2>
>>>
>>> Em ter, 13 de ago de 2019 às 19:29, Jeferson Almir <
>>> jefersonram...@gmail.com> escreveu:
>>>
>>>> Como eu provo que não existem 2 naturais cuja soma e diferença de seus
>>>> quadrados sejam quadrados ?
>>>>
>>>> Ps: eu tentei pegar a solução clássica da equação da soma x^2 + y^2 =
>>>> z^2 e tentei jogar na diferença pra aparecer algum absurdo em algum módulo
>>>> mas obtive sucesso.
>>>>
>>>> --
>>>> Esta mensagem foi verificada pelo sistema de antivírus e
>>>> acredita-se estar livre de perigo.
>>>
>>>
>>>
>>> --
>>> Israel Meireles Chrisostomo
>>>
>>
>>
>> --
>> Israel Meireles Chrisostomo
>>
>> --
>> Esta mensagem foi verificada pelo sistema de antivírus e
>> acredita-se estar livre de perigo.
>
>
> --
> Esta mensagem foi verificada pelo sistema de antiv�rus e
> acredita-se estar livre de perigo.
> --
> Esta mensagem foi verificada pelo sistema de antivírus e
> acredita-se estar livre de perigo.
>

-- 
Esta mensagem foi verificada pelo sistema de antiv�rus e
 acredita-se estar livre de perigo.



Re: [obm-l] Triplas pitagoricas

2019-08-15 Por tôpico Jeferson Almir
Obrigado a todos pelas ideias apresentadas.

Em qua, 14 de ago de 2019 às 17:13, Israel Meireles Chrisostomo <
israelmchrisost...@gmail.com> escreveu:

>
> Sejam x²+y²=z² e x²-y²=z'² subtraindo as duas igualdades temos:
> 2y²=z²-z'²
> Suponha sem perda de generalidade que z'= z-c daí então teremos:
>
> 2y²=z²-(z-c)²=2cz-c²
>
> Daí então segue que:
>
> 2y²=c(2z-c)
> Temos duas possibilidades ou c=2m e daí então
> y²=m(2z-c) o que implica que m=2z-c-> 2m=4z-2c->c=z.
> Por outro lado se 2z-c=2m daí então y²=mc-> m=c->2m=2c
> 2z-c=2c->z=3c/2 substituindo os valores nota-se que não há como ser
> satisfeito
>
> <http://www.avg.com/email-signature?utm_medium=email_source=link_campaign=sig-email_content=webmail>
>  Livre
> de vírus. www.avg.com
> <http://www.avg.com/email-signature?utm_medium=email_source=link_campaign=sig-email_content=webmail>.
>
> <#m_1622006173523811363_m_-8139557032871042333_DAB4FAD8-2DD7-40BB-A1B8-4E2AA1F9FDF2>
>
> Em ter, 13 de ago de 2019 às 19:48, Israel Meireles Chrisostomo <
> israelmchrisost...@gmail.com> escreveu:
>
>> opa me desculpe ju errei aqui desculpe -me
>>
>>
>> <http://www.avg.com/email-signature?utm_medium=email_source=link_campaign=sig-email_content=webmail>
>>  Livre
>> de vírus. www.avg.com
>> <http://www.avg.com/email-signature?utm_medium=email_source=link_campaign=sig-email_content=webmail>.
>>
>> <#m_1622006173523811363_m_-8139557032871042333_m_9119343842984229474_DAB4FAD8-2DD7-40BB-A1B8-4E2AA1F9FDF2>
>>
>> Em ter, 13 de ago de 2019 às 19:46, Israel Meireles Chrisostomo <
>> israelmchrisost...@gmail.com> escreveu:
>>
>>> Suponha sem perda de generalidade que x,y,z são positivos. Vc tem x^2 +
>>> y^2 = z^2 e x^2 - y^2 = z^2 somando as duas equações temos x^2=z^2 e então
>>> x=z por outro lado subtraindo as duas igualdades y^2=z^2 o que implica que
>>> y=z isso implica que 2x^2=x^2 e então 2=1 o que é um absurdo
>>>
>>>
>>> <http://www.avg.com/email-signature?utm_medium=email_source=link_campaign=sig-email_content=webmail>
>>>  Livre
>>> de vírus. www.avg.com
>>> <http://www.avg.com/email-signature?utm_medium=email_source=link_campaign=sig-email_content=webmail>.
>>>
>>> <#m_1622006173523811363_m_-8139557032871042333_m_9119343842984229474_m_-315779286925050189_DAB4FAD8-2DD7-40BB-A1B8-4E2AA1F9FDF2>
>>>
>>> Em ter, 13 de ago de 2019 às 19:29, Jeferson Almir <
>>> jefersonram...@gmail.com> escreveu:
>>>
>>>> Como eu provo que não existem 2 naturais cuja soma e diferença de seus
>>>> quadrados sejam quadrados ?
>>>>
>>>> Ps: eu tentei pegar a solução clássica da equação da soma x^2 + y^2 =
>>>> z^2 e tentei jogar na diferença pra aparecer algum absurdo em algum módulo
>>>> mas obtive sucesso.
>>>>
>>>> --
>>>> Esta mensagem foi verificada pelo sistema de antivírus e
>>>> acredita-se estar livre de perigo.
>>>
>>>
>>>
>>> --
>>> Israel Meireles Chrisostomo
>>>
>>
>>
>> --
>> Israel Meireles Chrisostomo
>>
>
>
> --
> Israel Meireles Chrisostomo
>
> --
> Esta mensagem foi verificada pelo sistema de antivírus e
> acredita-se estar livre de perigo.

-- 
Esta mensagem foi verificada pelo sistema de antiv�rus e
 acredita-se estar livre de perigo.



Re: [obm-l] Triplas pitagoricas

2019-08-13 Por tôpico Jeferson Almir
Tem que ser algo do tipo Israel
x^2 + y^2 = A^2
x^2 - y^2 = B^2

Em ter, 13 de ago de 2019 às 19:56, Israel Meireles Chrisostomo <
israelmchrisost...@gmail.com> escreveu:

> opa me desculpe ju errei aqui desculpe -me
>
>
> <http://www.avg.com/email-signature?utm_medium=email_source=link_campaign=sig-email_content=webmail>
>  Livre
> de vírus. www.avg.com
> <http://www.avg.com/email-signature?utm_medium=email_source=link_campaign=sig-email_content=webmail>.
> <#m_-7862083770974194334_DAB4FAD8-2DD7-40BB-A1B8-4E2AA1F9FDF2>
>
> Em ter, 13 de ago de 2019 às 19:46, Israel Meireles Chrisostomo <
> israelmchrisost...@gmail.com> escreveu:
>
>> Suponha sem perda de generalidade que x,y,z são positivos. Vc tem x^2 +
>> y^2 = z^2 e x^2 - y^2 = z^2 somando as duas equações temos x^2=z^2 e então
>> x=z por outro lado subtraindo as duas igualdades y^2=z^2 o que implica que
>> y=z isso implica que 2x^2=x^2 e então 2=1 o que é um absurdo
>>
>>
>> <http://www.avg.com/email-signature?utm_medium=email_source=link_campaign=sig-email_content=webmail>
>>  Livre
>> de vírus. www.avg.com
>> <http://www.avg.com/email-signature?utm_medium=email_source=link_campaign=sig-email_content=webmail>.
>>
>> <#m_-7862083770974194334_m_-315779286925050189_DAB4FAD8-2DD7-40BB-A1B8-4E2AA1F9FDF2>
>>
>> Em ter, 13 de ago de 2019 às 19:29, Jeferson Almir <
>> jefersonram...@gmail.com> escreveu:
>>
>>> Como eu provo que não existem 2 naturais cuja soma e diferença de seus
>>> quadrados sejam quadrados ?
>>>
>>> Ps: eu tentei pegar a solução clássica da equação da soma x^2 + y^2 =
>>> z^2 e tentei jogar na diferença pra aparecer algum absurdo em algum módulo
>>> mas obtive sucesso.
>>>
>>> --
>>> Esta mensagem foi verificada pelo sistema de antivírus e
>>> acredita-se estar livre de perigo.
>>
>>
>>
>> --
>> Israel Meireles Chrisostomo
>>
>
>
> --
> Israel Meireles Chrisostomo
>
> --
> Esta mensagem foi verificada pelo sistema de antivírus e
> acredita-se estar livre de perigo.

-- 
Esta mensagem foi verificada pelo sistema de antiv�rus e
 acredita-se estar livre de perigo.



[obm-l] Triplas pitagoricas

2019-08-13 Por tôpico Jeferson Almir
Como eu provo que não existem 2 naturais cuja soma e diferença de seus
quadrados sejam quadrados ?

Ps: eu tentei pegar a solução clássica da equação da soma x^2 + y^2 = z^2 e
tentei jogar na diferença pra aparecer algum absurdo em algum módulo mas
obtive sucesso.

-- 
Esta mensagem foi verificada pelo sistema de antiv�rus e
 acredita-se estar livre de perigo.



[obm-l] Torneio de Casais

2019-07-17 Por tôpico Jeferson Almir
Olá amigos peço uma ajuda no seguinte problema.

Em um grupo há H homens e M mulheres. Sabe-se que M é maior ou igual a 2H -
1. Prove que é possível organizar uma dança em pares tal que todos os
homens dançam e todo homem que não conhece a mulher com quem fez par,
conhece apenas mulheres que não dançam.

-- 
Esta mensagem foi verificada pelo sistema de antiv�rus e
 acredita-se estar livre de perigo.



[obm-l] Re: [obm-l] Re: [obm-l] Pontuação de Campeonato

2019-06-07 Por tôpico Jeferson Almir
Valeu Ralph !!
Só pra terminar eu precisaria exibir uma tabela que 33 pontos não é
suficiente??? Abraço.

Em sex, 7 de jun de 2019 às 00:22, Ralph Teixeira 
escreveu:

> RESPOSTA: 34 pontos.
>
> Quando o campeonato termina, os 17 melhores times jogaram 17x16/2=17x8
> partidas entre si, mais 17x3 partidas com os ultimos 3 times. Assim, esses
> 17 "melhores" times tem acesso a, no maximo, 17x11x3 pontos, ou seja, 33
> pontos cada um na media (no maximo!).
>
> Isso significa que, se voce faz 34 pontos, com certeza escapa do
> rebaixamento (eh impossivel que os 17 primeiros times tenham >=34 pontos
> cada).
>
> Isto dito, com MUITO azar pode ser que algum time com 33 pontos seja
> rebaixado. Basta imaginar que os 17 primeiros ganham TODOS os jogos dos 3
> outros, e entre si cada um dos 17 primeiros ganha 8 e perde 8 (para mostrar
> categoricamente que isso eh possivel, ponha os 17 times num circulo e
> imagine que cada time ganha dos 8 mais proximos no sentido horario, e perde
> dos outros 8). Entao voce tem uma tabela onde 17 times terminam com (cada
> um) 11 vitorias e 8 derrotas, ou seja, 33 pontos cada, e um deles seria
> rebaixado (em algum criterio de desempate).
>
> Abraco, Ralph.
>
> On Thu, Jun 6, 2019 at 10:44 PM Jeferson Almir 
> wrote:
>
>> Qual a pontuação mínima de um campeonato com 20 times  para que um time
>> fique livre do rebaixamento( 4 últimos times  descem ) sabendo que cada
>> time joga com todos os outros somente uma única vez??. E que vitória vale 3
>> pontos empate vale 1 ponto.
>>
>> --
>> Esta mensagem foi verificada pelo sistema de antivírus e
>> acredita-se estar livre de perigo.
>
>
> --
> Esta mensagem foi verificada pelo sistema de antivírus e
> acredita-se estar livre de perigo.

-- 
Esta mensagem foi verificada pelo sistema de antiv�rus e
 acredita-se estar livre de perigo.



[obm-l] Pontuação de Campeonato

2019-06-06 Por tôpico Jeferson Almir
Qual a pontuação mínima de um campeonato com 20 times  para que um time
fique livre do rebaixamento( 4 últimos times  descem ) sabendo que cada
time joga com todos os outros somente uma única vez??. E que vitória vale 3
pontos empate vale 1 ponto.

-- 
Esta mensagem foi verificada pelo sistema de antiv�rus e
 acredita-se estar livre de perigo.



Re: [obm-l] Probabilidade de Moedas

2019-05-27 Por tôpico Jeferson Almir
Puxa Ralph valeu demais!!

Em seg, 27 de mai de 2019 às 22:58, Ralph Teixeira 
escreveu:

> Ah, esse eh um problema classico e MUITO bonito! :D
>
> Seja A o evento: "Tem mais caras nas vermelhas do que coroas nas pretas."
> Seja B o evento: "Tem mais coroas nas vermelhas do que caras nas pretas."
>
> Queremos p(A). Note que p(A)=p(B) por simetria (moedas honestas, nada muda
> se trocar cara por coroa).
>
> Enfim, note que um, e apenas um dos dois eventos A e B acontece! De fato,
> sejam KV (e KP) o numero de caras vermelhas (e pretas) obtidas, CV (e CP) o
> numero de coroas vermelhas (e pretas). Em particular, CV+KV=n+1 e CP+KP=n.
> Assim:
>
> -- Se ambos A e B falhassem, teriamos KV<=CP e CV<=KP, portanto
> KV+CV<=KP+CP, absurdo.
> -- Se A e B ambos valessem, teriamos KV>CP e CV>KP, o que nos inteiros
> implica KV>=CP+1 e CV>=KP+1, ou seja KV+CV>=KP+CP+2, absurdo tambem!
>
> Em suma, p(A)=p(B)=1/2, ou seja, a resposta eh 50%.
>
> (Note que para ter esta solucao sem conta quase nenhuma, os numeros TEM
> QUE SER n+1 e n. Se fossem n moedas de cada, ou n+2 de uma e n da outra,
> teriamos que fazer umas contas mais complicadas...)
>
> Abraco, Ralph.
>
>
> On Mon, May 27, 2019 at 9:45 PM Jeferson Almir 
> wrote:
>
>> Dispomos de 2n+1 moedas honestas, sendo n+1 vermelhas e n pretas. Uma
>> pessoa arremessa as 2n+1 moedas simultaneamente, qual a probabilidade de se
>> obter MAIS caras de vermelhas do que coroas de pretas ?
>> Peço ajuda nesse problema.
>>
>> --
>> Esta mensagem foi verificada pelo sistema de antivírus e
>> acredita-se estar livre de perigo.
>
>
> --
> Esta mensagem foi verificada pelo sistema de antivírus e
> acredita-se estar livre de perigo.

-- 
Esta mensagem foi verificada pelo sistema de antiv�rus e
 acredita-se estar livre de perigo.



[obm-l] Probabilidade de Moedas

2019-05-27 Por tôpico Jeferson Almir
Dispomos de 2n+1 moedas honestas, sendo n+1 vermelhas e n pretas. Uma
pessoa arremessa as 2n+1 moedas simultaneamente, qual a probabilidade de se
obter MAIS caras de vermelhas do que coroas de pretas ?
Peço ajuda nesse problema.

-- 
Esta mensagem foi verificada pelo sistema de antiv�rus e
 acredita-se estar livre de perigo.



[obm-l] Re: [obm-l] Re: [obm-l] Re: [obm-l] Re: [obm-l] Re: [obm-l] Re: [obm-l] Polinômio Inteiro

2019-05-02 Por tôpico Jeferson Almir
Por nada Pedro !! E sen1º é um  número algébrico . Abraço.

Em qui, 2 de mai de 2019 às 10:52, Pedro José 
escreveu:

> Bom dia!
> Jeferson,
> obrigado! Pensava, na verdade tinha certeza que sen 1grau era
> transcendente.
> Fui até pesquisar o teorema d*e *Lindemann-Weierstrass*, *que nem me
> recordava o nome, mas é para sen1, mas não um grau e sim radiano.
> Falha de armazenamento na memória.
>
> Sds,
> PJMS
>
>
>
> Em qua, 1 de mai de 2019 às 06:46, Jeferson Almir <
> jefersonram...@gmail.com> escreveu:
>
>> Puxa Raph mais uma vez muito obrigado!!
>>
>> Em ter, 30 de abr de 2019 às 19:17, Ralph Teixeira 
>> escreveu:
>>
>>> Oi, Jeferson.
>>>
>>> Sua ideia funciona: comece com P(x,y)=(y+ix)^180+1. Como voce disse,
>>> P(s,c)=0 onde c=cos1º e s=sin1º.
>>>
>>> Agora olhemos para a parte real deste polinomio: ateh dah para escrever
>>> explicitamente, mas eu vou me limitar a dizer que eh algo do tipo
>>> R(x,y)=SOMA(a_k*y^(2k)*x^(180-2k))+1 onde os a_k sao coeficientes inteiros
>>> (que incluem os sinais negativos que porventura venham das potencias pares
>>> de i).
>>>
>>> Enfim, o truque eh perceber que todas as potencias de y ali sao pares.
>>> Ou seja, trocando y^2 por 1-x^2 em todos os termos, voce vai ficar com um
>>> polinomio P(x) que satisfaz o que voce quer.
>>>
>>> Abraco, Ralph.
>>>
>>> On Tue, Apr 30, 2019 at 6:02 PM Jeferson Almir 
>>> wrote:
>>>
>>>> Eu estou tentando através do binômio de Newton obter tal polinômio
>>>> pegando a parte real do número complexo. Sen1º não é transcende.
>>>>
>>>> Em ter, 30 de abr de 2019 às 17:35, Pedro José 
>>>> escreveu:
>>>>
>>>>> Boa tarde!
>>>>> Não compreendi
>>>>> sen1º é um número transcendente, ou não??
>>>>>
>>>>> Sds,
>>>>> PJMS
>>>>>
>>>>>
>>>>> Em ter, 30 de abr de 2019 às 14:30, Jeferson Almir <
>>>>> jefersonram...@gmail.com> escreveu:
>>>>>
>>>>>> Mostre que existe um polinômio P(x) de coeficientes inteiros que
>>>>>> possui sen1º como raiz de P(x).
>>>>>>
>>>>>>
>>>>>> Eu tentei usar a forma exponencial de números complexos (Euler)
>>>>>>  e^(i.pi/180) = cos1º + isen1º e depois elevando 180 e pegando a parte 
>>>>>> real
>>>>>> do complexo mas ainda não consegui .
>>>>>>
>>>>>> --
>>>>>> Esta mensagem foi verificada pelo sistema de antivírus e
>>>>>> acredita-se estar livre de perigo.
>>>>>
>>>>>
>>>>> --
>>>>> Esta mensagem foi verificada pelo sistema de antivírus e
>>>>> acredita-se estar livre de perigo.
>>>>
>>>>
>>>> --
>>>> Esta mensagem foi verificada pelo sistema de antivírus e
>>>> acredita-se estar livre de perigo.
>>>
>>>
>>> --
>>> Esta mensagem foi verificada pelo sistema de antivírus e
>>> acredita-se estar livre de perigo.
>>
>>
>> --
>> Esta mensagem foi verificada pelo sistema de antivírus e
>> acredita-se estar livre de perigo.
>
>
> --
> Esta mensagem foi verificada pelo sistema de antivírus e
> acredita-se estar livre de perigo.

-- 
Esta mensagem foi verificada pelo sistema de antiv�rus e
 acredita-se estar livre de perigo.



[obm-l] Re: [obm-l] Re: [obm-l] Re: [obm-l] Re: [obm-l] Polinômio Inteiro

2019-05-01 Por tôpico Jeferson Almir
Puxa Raph mais uma vez muito obrigado!!

Em ter, 30 de abr de 2019 às 19:17, Ralph Teixeira 
escreveu:

> Oi, Jeferson.
>
> Sua ideia funciona: comece com P(x,y)=(y+ix)^180+1. Como voce disse,
> P(s,c)=0 onde c=cos1º e s=sin1º.
>
> Agora olhemos para a parte real deste polinomio: ateh dah para escrever
> explicitamente, mas eu vou me limitar a dizer que eh algo do tipo
> R(x,y)=SOMA(a_k*y^(2k)*x^(180-2k))+1 onde os a_k sao coeficientes inteiros
> (que incluem os sinais negativos que porventura venham das potencias pares
> de i).
>
> Enfim, o truque eh perceber que todas as potencias de y ali sao pares. Ou
> seja, trocando y^2 por 1-x^2 em todos os termos, voce vai ficar com um
> polinomio P(x) que satisfaz o que voce quer.
>
> Abraco, Ralph.
>
> On Tue, Apr 30, 2019 at 6:02 PM Jeferson Almir 
> wrote:
>
>> Eu estou tentando através do binômio de Newton obter tal polinômio
>> pegando a parte real do número complexo. Sen1º não é transcende.
>>
>> Em ter, 30 de abr de 2019 às 17:35, Pedro José 
>> escreveu:
>>
>>> Boa tarde!
>>> Não compreendi
>>> sen1º é um número transcendente, ou não??
>>>
>>> Sds,
>>> PJMS
>>>
>>>
>>> Em ter, 30 de abr de 2019 às 14:30, Jeferson Almir <
>>> jefersonram...@gmail.com> escreveu:
>>>
>>>> Mostre que existe um polinômio P(x) de coeficientes inteiros que possui
>>>> sen1º como raiz de P(x).
>>>>
>>>>
>>>> Eu tentei usar a forma exponencial de números complexos (Euler)
>>>>  e^(i.pi/180) = cos1º + isen1º e depois elevando 180 e pegando a parte real
>>>> do complexo mas ainda não consegui .
>>>>
>>>> --
>>>> Esta mensagem foi verificada pelo sistema de antivírus e
>>>> acredita-se estar livre de perigo.
>>>
>>>
>>> --
>>> Esta mensagem foi verificada pelo sistema de antivírus e
>>> acredita-se estar livre de perigo.
>>
>>
>> --
>> Esta mensagem foi verificada pelo sistema de antivírus e
>> acredita-se estar livre de perigo.
>
>
> --
> Esta mensagem foi verificada pelo sistema de antivírus e
> acredita-se estar livre de perigo.

-- 
Esta mensagem foi verificada pelo sistema de antiv�rus e
 acredita-se estar livre de perigo.



[obm-l] Re: [obm-l] Re: [obm-l] Polinômio Inteiro

2019-04-30 Por tôpico Jeferson Almir
Eu estou tentando através do binômio de Newton obter tal polinômio pegando
a parte real do número complexo. Sen1º não é transcende.

Em ter, 30 de abr de 2019 às 17:35, Pedro José 
escreveu:

> Boa tarde!
> Não compreendi
> sen1º é um número transcendente, ou não??
>
> Sds,
> PJMS
>
>
> Em ter, 30 de abr de 2019 às 14:30, Jeferson Almir <
> jefersonram...@gmail.com> escreveu:
>
>> Mostre que existe um polinômio P(x) de coeficientes inteiros que possui
>> sen1º como raiz de P(x).
>>
>>
>> Eu tentei usar a forma exponencial de números complexos (Euler)
>>  e^(i.pi/180) = cos1º + isen1º e depois elevando 180 e pegando a parte real
>> do complexo mas ainda não consegui .
>>
>> --
>> Esta mensagem foi verificada pelo sistema de antivírus e
>> acredita-se estar livre de perigo.
>
>
> --
> Esta mensagem foi verificada pelo sistema de antivírus e
> acredita-se estar livre de perigo.

-- 
Esta mensagem foi verificada pelo sistema de antiv�rus e
 acredita-se estar livre de perigo.



[obm-l] Polinômio Inteiro

2019-04-30 Por tôpico Jeferson Almir
Mostre que existe um polinômio P(x) de coeficientes inteiros que possui
sen1º como raiz de P(x).


Eu tentei usar a forma exponencial de números complexos (Euler)
 e^(i.pi/180) = cos1º + isen1º e depois elevando 180 e pegando a parte real
do complexo mas ainda não consegui .

-- 
Esta mensagem foi verificada pelo sistema de antiv�rus e
 acredita-se estar livre de perigo.



[obm-l] Re: [obm-l] Re: [obm-l] Palíndromos Perfeitos de 4 dígitos

2019-03-31 Por tôpico Jeferson Almir
Seja (ABCD) de quatro dígitos queremos saber se a sua raiz quadrada produz
coisas do tipo
(ABCD)^1/2 = XY então ( DCBA)^1/2= YX,  que foi justamente a sua
interpretação.

Em dom, 31 de mar de 2019 às 21:15, Pedro José 
escreveu:

> Boa noite!
> Envio espúrio;
> Continuando
> Seja X=5 ==> A = 2 ou 3  ==> (Y5)^2 = (BCDA) absurdo, deveria acabar em 5
> Seja X=6 ==> A = 3 ou 4 ==> (Y6)^2 = (BCDA) absurdo, deveria acabar em 6
> Seja X=7 ==> A = 5 ou 6 ==> (Y7)^2 = (BCDA) absurdo, deveria acabar em 9
> Seja X=8 ==> A = 6 ou 7  ==> (Y8)^2 = (BCDA) absurdo, deveria acabar em 4
> Seja X=9 ==> A = 8 ou 9 ==> (Y9)^2 = (BCDA) absurdo, deveria acabar em 1
>
> Portanto, não há solução para o proposto, se o entendimento do enunciado
> estiver correto
>
> Saudações,
> PJMS.
>
>
> Em dom, 31 de mar de 2019 às 21:00, Pedro José 
> escreveu:
>
>> Boa noite!
>> Não sei se compreendi bem o enunciado.
>> Dado um quadrado de um número de dois dígitos XY, com X, Y sendo
>> algarismos cujo número que eles representam (X), (y) >=3  formado pelos
>> dígitos ABCD a raiz de  (DABC) = (YX) ou raiz de (BCDA) = (YX)   onde (XY)
>> significa concatenação dos algarissmos XY e assim por diante.
>> Vamos ver a permutação circular (DABC)
>> Seja Y = 9  ==> D = 1 ==> (DABC) < 2.000 e (DABC) = (9X)^2 >= 8.100
>> absurdo.
>> Seja Y= 8 ==> D=4 ==> (DABC) < 5.000 e (DABC)=(8X)^2>= 6.400 absurdo.
>> Seja Y= 7 ==> D=9 ==> (DABC) > 9.000 e (DABC)= (7X)^2 < 6.400 absurdo.
>> Seja Y = 6 ==> D = 6 ==> (DABC)> 6.000 e (DABC)= (6X)^2 < 4.900 absurdo.
>> Seja Y = 5 ==> D = 5 ==> (DABC)> 5.000 e (DABC)= (5X)^2 < 3.600 absurdo.
>> Seja Y = 4 ==> D = 6 ==> (DABC)> 6.000 e (DABC)= (4X)^2 < 2.500 absurdo.
>> Seja Y = 3 ==> D = 9 ==> (DABC)> 9.000 e (DABC)= (3X)^2 <1.600 absurdo.
>> Portanto não há como atender essa rotação veremos a outra.
>>
>> Seja X=3 ==> A = 1 ==> (Y3)^2 = (BCD1) absurdo, deveria acabar em 9
>> Seja X=4 ==> A = 1 ou A=2 ==> (Y4)^2 = (BCDA) absurdo, deveria acabar em 6
>> Seja X=5 ==> A = 2 ou 3 ==> (Y3)^2 = (BCD1) absurdo, deveria acabar em 9
>>
>>
>>
>>
>> Em qui, 28 de mar de 2019 às 16:23, Jeferson Almir <
>> jefersonram...@gmail.com> escreveu:
>>
>>> Um número é palíndromo perfeito se satisfaz a relação que segue:
>>>
>>> (144)^1/2 = 12 e (441)^1/2 = 21
>>>
>>> (169)^1/2 = 13  e (961)^1/2
>>> <https://maps.google.com/?q=e+(961)%5E1/2=gmail=g> = 31
>>>
>>> Determinar os palíndromos perfeitos de  4 dígitos.
>>>
>>> --
>>> Esta mensagem foi verificada pelo sistema de antivírus e
>>> acredita-se estar livre de perigo.
>>
>>
> --
> Esta mensagem foi verificada pelo sistema de antivírus e
> acredita-se estar livre de perigo.

-- 
Esta mensagem foi verificada pelo sistema de antiv�rus e
 acredita-se estar livre de perigo.



[obm-l] Palíndromos Perfeitos de 4 dígitos

2019-03-28 Por tôpico Jeferson Almir
Um número é palíndromo perfeito se satisfaz a relação que segue:

(144)^1/2 = 12 e (441)^1/2 = 21

(169)^1/2 = 13  e (961)^1/2 = 31

Determinar os palíndromos perfeitos de  4 dígitos.

-- 
Esta mensagem foi verificada pelo sistema de antiv�rus e
 acredita-se estar livre de perigo.



[obm-l] Fibonacci terminado em zeros

2019-03-24 Por tôpico Jeferson Almir
Como eu provo que existe um Fibonacci terminado em n zeros ?

-- 
Esta mensagem foi verificada pelo sistema de antiv�rus e
 acredita-se estar livre de perigo.



[obm-l] Torneio das Cidades ( Número mínimo de Tentativas )

2019-02-24 Por tôpico Jeferson Almir
Peço ajuda aos amigos da lista, sei que existe um problemas da obm
"parecido",  aguardo dicas ou soluções. Eu tentei formar um grafo de
tentativas e penso como otimizar ele.

a.) Existem 2n + 1 (n> 2) baterias. Não sabemos quais baterias são boas e
quais são ruins, mas sabemos que o número de baterias boas é maior do que o
número de baterias ruins. Uma lâmpada usa duas baterias e só funciona se
ambas forem boas. Qual é o menor número de tentativas suficientes para
fazer a lâmpada funcionar?

b.) O mesmo problema, mas o número total de baterias é 2n (n> 2) e os
números de baterias boas e ruins são iguais.

-- 
Esta mensagem foi verificada pelo sistema de antiv�rus e
 acredita-se estar livre de perigo.



[obm-l] Re: [obm-l] Re: [obm-l] Re: [obm-l] Sequência de Fibonacci

2019-02-14 Por tôpico Jeferson Almir
Valeu Ralph!! Suas ideias Phodas sempre salvando o dia !!

Em qui, 14 de fev de 2019 às 12:36, Israel Meireles Chrisostomo <
israelmchrisost...@gmail.com> escreveu:

> caramba ralph, quanta engenhosidade!!!
>
>
> --
> Esta mensagem foi verificada pelo sistema de antivírus e
> acredita-se estar livre de perigo.

-- 
Esta mensagem foi verificada pelo sistema de antiv�rus e
 acredita-se estar livre de perigo.



[obm-l] Recorrência de 2ª Ordem

2019-02-14 Por tôpico Jeferson Almir
Olá companheiros da lista, quando nos deparamos com uma recorrência de
segunda ordem e na tentativa de resolvê-la montamos um equação ou polinômio
característico, e minha dúvida está em  saber como deduzir a solução da
equação de recorrência para o caso em que as raizes são iguaispois o
caso em que ela são diferentes eu consegui, se vc jogar a solução x = (A +
Bn)(lambda)^n dá perfeito  mas como chegar nela ?

-- 
Esta mensagem foi verificada pelo sistema de antiv�rus e
 acredita-se estar livre de perigo.



[obm-l] Sequência de Fibonacci

2019-02-13 Por tôpico Jeferson Almir
Como provar esse resultado de fibonacci que não seja por indução ??
F_2m •F_m-1 - F_2m-1•F_m = (-1)^m•F_m

-- 
Esta mensagem foi verificada pelo sistema de antiv�rus e
 acredita-se estar livre de perigo.



Re: [obm-l] Divisibilidade por 13 e 19

2019-02-10 Por tôpico Jeferson Almir
Muito obrigado senhores!!

Em dom, 10 de fev de 2019 às 22:09, Artur Steiner <
artur.costa.stei...@gmail.com> escreveu:

> Há um critério que conheço, mas em termos práticos não serve pra nada. É
> melhor fazer a divisão.
>
> No caso de 13, vc toma 10 - 13 = -3 e, na representação decimal do número,
> substitui 10 por -3 e faz as contas. O número é divisível por 13 se, é
> somente se, o resultado for divisível por 13. Analogamente para 19. Vale
> qualquer que seja o número de algarismos.
>
> Por exemplo, o número 156. Calculamos 1 x (-3)^2 + 5 x (-3) + 6 = 0,
> divisível por 13. Logo, 156 é divisível por 13.
>
> Agora, 209. Obtemos 2 x (-9)^2 × 0 x (-9) + 9 = 162 + 9 = 171 = 9 x 19. E
> 209 é divisível por 19.
>
> É o mesmo processo dos famosos critérios de divisibilidade por 9 e por 11.
> E tem aquele semelhante para 3 porque 3^2 = 9.
>
> Pode ser provado pelas propriedades dos polinômios ou por congruências.
>
> Mas, no caso de 13, 19 e mesmo 7, em termos práticos, em nada facilita.
>
> Não sei se há um critério melhor.
>
>
>
> Artur Costa Steiner
>
> Em dom, 10 de fev de 2019 20:56, Jeferson Almir  escreveu:
>
>> Considere um número de 4 algarismos da forma 70J7
>>
>> i) quais o valores de J para que o número seja divisível por 13 ?
>>
>> ii ) quais os valores de J para que o número seja divisível por 19 ?
>>
>> Uma vez que eu não faço ideia  quais são  os critérios de divisibilidade
>> por 13 e por 19, o algoritmo da divisão resolveria de alguma forma esse
>> problema ?? Ou existe outra forma de fazer sem usar o critério ???
>>
>> --
>> Esta mensagem foi verificada pelo sistema de antivírus e
>> acredita-se estar livre de perigo.
>
>
> --
> Esta mensagem foi verificada pelo sistema de antivírus e
> acredita-se estar livre de perigo.

-- 
Esta mensagem foi verificada pelo sistema de antiv�rus e
 acredita-se estar livre de perigo.



[obm-l] Divisibilidade por 13 e 19

2019-02-10 Por tôpico Jeferson Almir
Considere um número de 4 algarismos da forma 70J7

i) quais o valores de J para que o número seja divisível por 13 ?

ii ) quais os valores de J para que o número seja divisível por 19 ?

Uma vez que eu não faço ideia  quais são  os critérios de divisibilidade
por 13 e por 19, o algoritmo da divisão resolveria de alguma forma esse
problema ?? Ou existe outra forma de fazer sem usar o critério ???

-- 
Esta mensagem foi verificada pelo sistema de antiv�rus e
 acredita-se estar livre de perigo.



[obm-l] OBM 2002 - Problema 6

2019-01-12 Por tôpico Jeferson Almir
Amigos peço ajuda nesse problema, e me orientaram a estudar Códigos
Corretores de Erros.


Arnaldo e Beatriz se comunicam durante um acampamento usando sinais de
fumaça, às vezes usando uma nuvem grande, às vezes uma pequena.

No tempo disponível antes do café da manhã, Arnaldo consegue enviar uma
seqüência de 24 nuvens. Como Beatriz nem sempre consegue distinguir uma
nuvem pequena de uma grande, ela e Arnaldo fizeram um dicionário antes de
ir para o acampamento. No dicionário aparecem N seqüências de 24 tamanhos
de nuvem (como por exemplo a seqüência PGPGPGPGPGPGGPGPGPGPGPGP, onde G
significa nuvem grande e P significa nuvem pequena). Para cada uma das N
seqüências, o dicionário indica seu significado. Para evitar interpretações
erradas, Arnaldo e Beatriz evitaram incluir no dicionário seqüências
parecidas. Mais precisamente, duas seqüências no dicionário sempre diferem
em pelo menos 8 das 24 posições.



Demonstre que N<= 4096

-- 
Esta mensagem foi verificada pelo sistema de antiv�rus e
 acredita-se estar livre de perigo.



Re: [obm-l] Problema 6 - IMO 2001

2018-11-14 Por tôpico Jeferson Almir
A ordem segue a,d,b,c no sentido horário devido a relação a^2 -ac + c^2 =
b^2 + bd + d^2

Em qua, 14 de nov de 2018 às 15:53, Pedro José 
escreveu:

> Boa tarde!
>
> Porém, me ficou uma dúvida! Como definir a ordem dos lados, os de medidas
> a e c devem ser adjacentes, assim como os de medida b e d.
> Mas como definir se os de a e b ou de a e d são adjacentes???
>
> Grato,
> PJMS
>
> Em ter, 13 de nov de 2018 às 13:44, Pedro José 
> escreveu:
>
>> Bom dia!
>>
>> Depois da observação do Anderson Torres é que atinei o quanto é bonita a
>> sua solução se você prosseguir.
>> Sua preocupação não deve ser em relação ao produto AC*BD, nem com os
>> valores AC ou BD; mas sim que tanto BD^2, como AC^2 são inteiros.
>> Falta uma beirinha e a solução indicada pelo Cláudio tem a dica final
>> Pelo menos para o caminho que vislumbrei.
>>
>> Saudações,
>> PJMS.
>>
>>
>>
>>
>> Em seg, 12 de nov de 2018 às 16:39, Anderson Torres <
>> torres.anderson...@gmail.com> escreveu:
>>
>>> Você quase resolveu! Posso dizer que esta era basicamente a solução
>>> oficial. Tente mais um pouco, que o caminho é esse.
>>>
>>> Em 8 de nov de 2018 23:27, "Jeferson Almir" 
>>> escreveu:
>>>
>>> Pessoal peço ajuda  no problema :
>>>
>>> Sejam a, b , c , d inteiros e a > b > c > d > 0 .
>>> Suponha que
>>> ac + bd = ( b+ d + a - c )( b+ d -a + c )
>>>
>>> Mostre que ab + cd não é primo .
>>>
>>>
>>> A minha ideia foi:
>>>
>>> Abrindo a relação de cima temos
>>>
>>> a^2 -ac + c^2 = b^2 + bd + d^2
>>>
>>> Então motivado pela ideia de usar geometria que um amigo falou fiz a
>>> suposição que temos um quadrilátero de lados a, d,b e c respectivamente e
>>>  nessa ultima relação usando lei dos cossenos teríamos A = 60° e C = 120°
>>> concluindo então que ABCD é inscritível . Aplicando Ptolomeu temos que 
>>> ACxBD=
>>> ab + cd e usando desigualdade triangular podemos afirmar que AC e BD não
>>> podem ser 1 . Mas ainda tem a possibilidade AC e BD  serem racionais !!
>>> Como provar que não podem ser ???
>>>
>>> --
>>> Esta mensagem foi verificada pelo sistema de antivírus e
>>> acredita-se estar livre de perigo.
>>>
>>>
>>>
>>> --
>>> Esta mensagem foi verificada pelo sistema de antivírus e
>>> acredita-se estar livre de perigo.
>>
>>
> --
> Esta mensagem foi verificada pelo sistema de antivírus e
> acredita-se estar livre de perigo.

-- 
Esta mensagem foi verificada pelo sistema de antiv�rus e
 acredita-se estar livre de perigo.



[obm-l] Expansão Multinomial

2018-11-14 Por tôpico Jeferson Almir
 Olá colegas da lista!!
Qual o  argumento combinatório para obter  o coeficiente   do termo x^k  de
uma expansão do tipo
(  1 + x + x^2 )^n. ??

-- 
Esta mensagem foi verificada pelo sistema de antiv�rus e
 acredita-se estar livre de perigo.



[obm-l] Problema 6 - IMO 2001

2018-11-08 Por tôpico Jeferson Almir
Pessoal peço ajuda  no problema :

Sejam a, b , c , d inteiros e a > b > c > d > 0 .
Suponha que
ac + bd = ( b+ d + a - c )( b+ d -a + c )

Mostre que ab + cd não é primo .


A minha ideia foi:

Abrindo a relação de cima temos

a^2 -ac + c^2 = b^2 + bd + d^2

Então motivado pela ideia de usar geometria que um amigo falou fiz a
suposição que temos um quadrilátero de lados a, d,b e c respectivamente e
 nessa ultima relação usando lei dos cossenos teríamos A = 60° e C = 120°
concluindo então que ABCD é inscritível . Aplicando Ptolomeu temos que ACxBD=
ab + cd e usando desigualdade triangular podemos afirmar que AC e BD não
podem ser 1 . Mas ainda tem a possibilidade AC e BD  serem racionais !!
Como provar que não podem ser ???

-- 
Esta mensagem foi verificada pelo sistema de antiv�rus e
 acredita-se estar livre de perigo.



{Disarmed} [obm-l] {Disarmed} Problema 6 - IMO 2001

2018-11-08 Por tôpico Jeferson Almir
Pessoal peço ajuda  no problema :

Sejam a, b , c , d inteiros e a > b > c > d > 0 .
Suponha que
ac + bd = ( b+ d + a - c )( b+ d -a + c )

Mostre que ab + cd não é primo .


A minha ideia foi:

Abrindo a relação de cima temos

a^2 -ac + c^2 = b^2 + bd + d^2

Então motivado pela ideia de usar geometria que um amigo falou fiz a
suposição que temos um quadrilátero de lados a, d,b e c respectivamente e
 nessa ultima relação usando lei dos cossenos teríamos A = 60° e C = 120°
concluindo então que ABCD é inscritível . Aplicando Ptolomeu temos que AC.BD
= ab + cd e usando desigualdade triangular podemos afirmar que AC e BD não
podem ser 1 . Mas ainda tem a possibilidade AC e BD  serem racionais !!
Como provar que não podem ser ???

-- 
Esta mensagem foi verificada pelo sistema de antiv�rus e
 acredita-se estar livre de perigo.



[obm-l] Re: [obm-l] Re: [obm-l] Re: Polinômios OBM 2015

2018-10-04 Por tôpico Jeferson Almir
Não !! Se não fui claro aqui vou mais uma vez!!
Quando eu pego 2 polinômios P(x) e Q(x) inteiros e o grau de P(x) é maior
que Q(x) e Q(x) é mônico,   então  o resto R(x)  da divisão será de
coeficientes inteiros. Eu não sei se de alguma forma por indução sai ou se
existe algum critério de irredutibilidade que garanta isso.

Em ter, 2 de out de 2018 às 13:56, Pedro José 
escreveu:

> Boa tarde!
>
> Não seria,:
>
> ...como eu provo que existe um?
> quando dividido por um polinômio mônico, de grau n e coeficientes
> racionais, nem todos inteiros?
>
> Saudações,
> PJMS
>
> Em ter, 2 de out de 2018 às 11:54, Jeferson Almir <
> jefersonram...@gmail.com> escreveu:
>
>> Amigos como eu provo que se um polinômio de coeficientes inteiros de grau
>> maior que  n+1 quando didivido por um polinômio mônico  de grau n e
>> coeficientes inteiros deixará um resto que é um polinômio de coeficientes
>> inteiros?? Isso resolveria o problema que peço ajuda
>>
>> Em sáb, 29 de set de 2018 às 00:18, Jeferson Almir <
>> jefersonram...@gmail.com> escreveu:
>>
>>> Peço ajuda no seguinte problema
>>>
>>> É verdade que existem um polinômio *f*(*x*) de coeficientes racionais,
>>> nem todos inteiros, de grau *n* > 0, um polinômio *g*(*x*), com todos
>>> os coeficientes inteiros, e um conjunto S com *n* + 1 inteiros tais que
>>> *g*(*t*) = *f*(*t*) para todo *t* pertencente  a S?
>>>
>>> *Minha idéia:* Eu fiz h(t) = g(t)- f(t) então h(t) tem grau maior ou
>>> igual a n+1 senão g(t) = f(t) o que é um absurdo pois g(t) tem coeficientes
>>> inteiros e f(t) não !! E quero provar que esse h(t) tem todos coeficientes
>>> inteiros através da forma fatorada das raízes mas estou conseguindo.
>>>
>>>
>>>
>> --
>> Esta mensagem foi verificada pelo sistema de antivírus e
>> acredita-se estar livre de perigo.
>
>
> --
> Esta mensagem foi verificada pelo sistema de antivírus e
> acredita-se estar livre de perigo.

-- 
Esta mensagem foi verificada pelo sistema de antiv�rus e
 acredita-se estar livre de perigo.



[obm-l] Re: Polinômios OBM 2015

2018-10-02 Por tôpico Jeferson Almir
Amigos como eu provo que se um polinômio de coeficientes inteiros de grau
maior que  n+1 quando didivido por um polinômio mônico  de grau n e
coeficientes inteiros deixará um resto que é um polinômio de coeficientes
inteiros?? Isso resolveria o problema que peço ajuda

Em sáb, 29 de set de 2018 às 00:18, Jeferson Almir 
escreveu:

> Peço ajuda no seguinte problema
>
> É verdade que existem um polinômio *f*(*x*) de coeficientes racionais,
> nem todos inteiros, de grau *n* > 0, um polinômio *g*(*x*), com todos os
> coeficientes inteiros, e um conjunto S com *n* + 1 inteiros tais que *g*(
> *t*) = *f*(*t*) para todo *t* pertencente  a S?
>
> *Minha idéia:* Eu fiz h(t) = g(t)- f(t) então h(t) tem grau maior ou
> igual a n+1 senão g(t) = f(t) o que é um absurdo pois g(t) tem coeficientes
> inteiros e f(t) não !! E quero provar que esse h(t) tem todos coeficientes
> inteiros através da forma fatorada das raízes mas estou conseguindo.
>
>
>

-- 
Esta mensagem foi verificada pelo sistema de antiv�rus e
 acredita-se estar livre de perigo.



[obm-l] Polinômios OBM 2015

2018-09-28 Por tôpico Jeferson Almir
Peço ajuda no seguinte problema

É verdade que existem um polinômio *f*(*x*) de coeficientes racionais, nem
todos inteiros, de grau *n* > 0, um polinômio *g*(*x*), com todos os
coeficientes inteiros, e um conjunto S com *n* + 1 inteiros tais que *g*(*t*)
= *f*(*t*) para todo *t* pertencente  a S?

*Minha idéia:* Eu fiz h(t) = g(t)- f(t) então h(t) tem grau maior ou igual
a n+1 senão g(t) = f(t) o que é um absurdo pois g(t) tem coeficientes
inteiros e f(t) não !! E quero provar que esse h(t) tem todos coeficientes
inteiros através da forma fatorada das raízes mas estou conseguindo.

-- 
Esta mensagem foi verificada pelo sistema de antiv�rus e
 acredita-se estar livre de perigo.



[obm-l] Re: [obm-l] Re: [obm-l] Polinômios ( RPM)

2018-09-21 Por tôpico Jeferson Almir
Valeu Esdras !!!

Em sex, 21 de set de 2018 às 01:40, Esdras Muniz 
escreveu:

> Suponha por absurdo que (7-Ri)>=0 para toda raiz Ri, i=1,...,100.
> Daí, por Ma>=Mg, temos:
> 1>=\sqer[100]{(7-R1)(7-R2)...(7-R100)}>1 então 1>1, o que é um absurdo.
>
> Em sex, 21 de set de 2018 às 01:05, Jeferson Almir <
> jefersonram...@gmail.com> escreveu:
>
>> Este problema é de uma R.P.M que não sei qual o exemplar e peço ajuda.
>>
>> Seja P(x) um polinômio de grau 100 tal que P(x) = x^100 -600x^99 +
>> 98x^98+97x^97 +... + a_1x + a_o tem 100 raizes reais e que P(7) > 1 .
>> Mostre que existe pelo menos uma raiz maior que 7 .
>>
>> Desconfio muito de usar médias mas não estou conseguindo adequar para
>> aplica-las .
>>
>> --
>> Esta mensagem foi verificada pelo sistema de antivírus e
>> acredita-se estar livre de perigo.
>
>
>
> --
> Esdras Muniz Mota
> Mestrando em Matemática
> Universidade Federal do Ceará
>
>
>
> --
> Esta mensagem foi verificada pelo sistema de antivírus e
> acredita-se estar livre de perigo.

-- 
Esta mensagem foi verificada pelo sistema de antiv�rus e
 acredita-se estar livre de perigo.



[obm-l] Polinômios ( RPM)

2018-09-20 Por tôpico Jeferson Almir
Este problema é de uma R.P.M que não sei qual o exemplar e peço ajuda.

Seja P(x) um polinômio de grau 100 tal que P(x) = x^100 -600x^99 +
98x^98+97x^97 +... + a_1x + a_o tem 100 raizes reais e que P(7) > 1 .
Mostre que existe pelo menos uma raiz maior que 7 .

Desconfio muito de usar médias mas não estou conseguindo adequar para
aplica-las .

-- 
Esta mensagem foi verificada pelo sistema de antiv�rus e
 acredita-se estar livre de perigo.



[obm-l] Equação Funcional

2018-09-19 Por tôpico Jeferson Almir
Peço uma ideia ou ajuda na seguinte questão:
Sejam x e y naturais e uma função  f : N -> N tais que
F(xy) = F(x) + F(y) -1

Existe um número finito de numeros tais que F(x) = 1.

F(30) = 4

Determine o F( 14400)

-- 
Esta mensagem foi verificada pelo sistema de antiv�rus e
 acredita-se estar livre de perigo.



[obm-l] Re: [obm-l] Re: [obm-l] Combinatória ( Semana Olímpica )

2018-06-25 Por tôpico Jeferson Almir
Valeu garoto !!!

Em seg, 25 de jun de 2018 às 09:32, Mauricio de Araujo <
mauricio.de.ara...@gmail.com> escreveu:

> Bom dia!!
>
> Este problema está discutido na página 52 do livro "de cuántas formas",
> cujo link coloco a seguir.
>
>
> https://drive.google.com/file/d/1TOu47F-UPUq9b0jr4sBwQ3I5Lnk6pxQg/view?usp=sharing
>
> Att.
> --
> Abraços,
> Mauricio de Araujo
> [oɾnɐɹɐ ǝp oıɔıɹnɐɯ]
>
>
> Em dom, 24 de jun de 2018 às 15:21, Jeferson Almir <
> jefersonram...@gmail.com> escreveu:
>
>> Peço ajuda nesse problema pois estou confuso em montar uma recorrência.
>>
>> Uma entrada de cinema custa 5 rands. Numa fila de 2n pessoas, há
>> exatamente n pessoas com notas de 5 rands e as outras n possuem notas de 10
>> rands. Inicialmente o caixa do cinema está vazio. De quantas maneiras
>> podemos organizar a fila de modo que o caixa sempre possa dar o troco?
>>
>> --
>> Esta mensagem foi verificada pelo sistema de antivírus e
>> acredita-se estar livre de perigo.
>
>
> --
> Esta mensagem foi verificada pelo sistema de antivírus e
> acredita-se estar livre de perigo.

-- 
Esta mensagem foi verificada pelo sistema de antiv�rus e
 acredita-se estar livre de perigo.



[obm-l] Combinatória ( Semana Olímpica )

2018-06-24 Por tôpico Jeferson Almir
Peço ajuda nesse problema pois estou confuso em montar uma recorrência.

Uma entrada de cinema custa 5 rands. Numa fila de 2n pessoas, há exatamente
n pessoas com notas de 5 rands e as outras n possuem notas de 10 rands.
Inicialmente o caixa do cinema está vazio. De quantas maneiras podemos
organizar a fila de modo que o caixa sempre possa dar o troco?

-- 
Esta mensagem foi verificada pelo sistema de antiv�rus e
 acredita-se estar livre de perigo.



[obm-l] Re: [obm-l] Re: [obm-l] Equação Funcional

2018-06-11 Por tôpico Jeferson Almir
Esse é o problema 2901 do livro Problemas Selecionados de Matemática (
Gandhi )
E resposta que ele diz é
R: x^3 - x^2 - 1 / x(x-1)

Em seg, 11 de jun de 2018 às 12:15, Jeferson Almir 
escreveu:

> Isso mesmo Ralph eu sei fazer g(x) = (x-1)/x
>
> Em seg, 11 de jun de 2018 às 11:33, Ralph Teixeira 
> escreveu:
>
>> Puxa, se fosse g(x)=(x-1)/x ali dentro do segundo termo, eu sabia fazer
>> rápido... :( Era só escrever y=g(x), z=g(y), e então:
>> f(x)+f(y)=1+x
>> f(y)+f(z)=1+y
>> f(z)+f(x)=1+z
>> pois é fácil ver que g(z)=g(g(g(x)))=x. Resolvendo esse sisteminha,
>> acharíamos f(x).
>>
>> Porém, com esse enunciado... Hm, alguém confere aqui o raciocínio abaixo,
>> porque acho que eu consigo mostrar que **não dá** para resolver isso, mas
>> estou morrendo de sono, então provavelmente escrevi alguma bobagem imensa.
>>
>> Observe que g(x)=(1-x)/x é injetiva (e sua inversa é g^(-1)(y)=1/(1+y)).
>> Dado um x_0=a, crie a sequência {x_k} com k inteiro onde x_(k+1)=g(x_k) --
>> observe que crio isto incluindo k negativo, o que é possível desde que
>> nenhum dos números da órbita seja 0 ou -1. Vou chamar o **conjunto** de
>> valores {x_k} de "órbita" do número a.
>>
>> Pois bem, a equação funcional só dá informações sobre os valores de f
>> dentro de cada órbita! Ela diz que f(x_k)+f(x_(k+1))=1+x_k (*), e mais
>> nada, ou seja, ela não relaciona os valores de f em órbitas distintas! Se a
>> órbita é infinita, isto é, se os x_k são todos distintos, você pode
>> ESCOLHER f(a) como quiser e calcular os outros f(x_k) usando (*) como
>> recorrência.
>>
>> Agora você me pergunta: porque a órbita não fecha? Bom, você tem razão,
>> para vários valores de "a" a órbita fecha, isto é, poderia ser x_P=x_0=a
>> para algum P<>0... Mas a equação x_P=a quer dizer g(g(g(...g(a))...)=a, que
>> é uma equação quadrática (né?), e portanto tem no máximo 2 raízes reais.
>> Então, mesmo que consideremos todos os P possíveis, o conjunto dos a que
>> fazem a órbita fechar é enumerável... Bom, os reais não são enumeráveis,
>> então há várias órbitas infinitas Acho.
>>
>> Abraço, Ralph.
>>
>> P.S.: Se eu tivesse bom senso, conferia isso antes de mandar para a
>> lista... Ah, dane-se, mesmo que eu esteja errado este tipo de raciocínio é
>> interessante, não?
>> P.S.2: Se o enunciado falar que f é *contínua*, aí talvez dê para fazer
>> algo usando o limite de x_k...
>>
>> On Mon, Jun 11, 2018 at 8:32 AM Jeferson Almir 
>> wrote:
>>
>>> Seja f(x) uma função real definida em R -{0,1} tal que
>>>
>>> f(x) + f( 1-x | x ) =1 + x determine f (x) .
>>>
>>> Obs:  ( 1-x | x) é 1-x dividido por x .
>>>
>>> --
>>> Esta mensagem foi verificada pelo sistema de antivírus e
>>> acredita-se estar livre de perigo.
>>
>>
>> --
>> Esta mensagem foi verificada pelo sistema de antivírus e
>> acredita-se estar livre de perigo.
>
>

-- 
Esta mensagem foi verificada pelo sistema de antiv�rus e
 acredita-se estar livre de perigo.



[obm-l] Re: [obm-l] Re: [obm-l] Equação Funcional

2018-06-11 Por tôpico Jeferson Almir
Isso mesmo Ralph eu sei fazer g(x) = (x-1)/x

Em seg, 11 de jun de 2018 às 11:33, Ralph Teixeira 
escreveu:

> Puxa, se fosse g(x)=(x-1)/x ali dentro do segundo termo, eu sabia fazer
> rápido... :( Era só escrever y=g(x), z=g(y), e então:
> f(x)+f(y)=1+x
> f(y)+f(z)=1+y
> f(z)+f(x)=1+z
> pois é fácil ver que g(z)=g(g(g(x)))=x. Resolvendo esse sisteminha,
> acharíamos f(x).
>
> Porém, com esse enunciado... Hm, alguém confere aqui o raciocínio abaixo,
> porque acho que eu consigo mostrar que **não dá** para resolver isso, mas
> estou morrendo de sono, então provavelmente escrevi alguma bobagem imensa.
>
> Observe que g(x)=(1-x)/x é injetiva (e sua inversa é g^(-1)(y)=1/(1+y)).
> Dado um x_0=a, crie a sequência {x_k} com k inteiro onde x_(k+1)=g(x_k) --
> observe que crio isto incluindo k negativo, o que é possível desde que
> nenhum dos números da órbita seja 0 ou -1. Vou chamar o **conjunto** de
> valores {x_k} de "órbita" do número a.
>
> Pois bem, a equação funcional só dá informações sobre os valores de f
> dentro de cada órbita! Ela diz que f(x_k)+f(x_(k+1))=1+x_k (*), e mais
> nada, ou seja, ela não relaciona os valores de f em órbitas distintas! Se a
> órbita é infinita, isto é, se os x_k são todos distintos, você pode
> ESCOLHER f(a) como quiser e calcular os outros f(x_k) usando (*) como
> recorrência.
>
> Agora você me pergunta: porque a órbita não fecha? Bom, você tem razão,
> para vários valores de "a" a órbita fecha, isto é, poderia ser x_P=x_0=a
> para algum P<>0... Mas a equação x_P=a quer dizer g(g(g(...g(a))...)=a, que
> é uma equação quadrática (né?), e portanto tem no máximo 2 raízes reais.
> Então, mesmo que consideremos todos os P possíveis, o conjunto dos a que
> fazem a órbita fechar é enumerável... Bom, os reais não são enumeráveis,
> então há várias órbitas infinitas Acho.
>
> Abraço, Ralph.
>
> P.S.: Se eu tivesse bom senso, conferia isso antes de mandar para a
> lista... Ah, dane-se, mesmo que eu esteja errado este tipo de raciocínio é
> interessante, não?
> P.S.2: Se o enunciado falar que f é *contínua*, aí talvez dê para fazer
> algo usando o limite de x_k...
>
> On Mon, Jun 11, 2018 at 8:32 AM Jeferson Almir 
> wrote:
>
>> Seja f(x) uma função real definida em R -{0,1} tal que
>>
>> f(x) + f( 1-x | x ) =1 + x determine f (x) .
>>
>> Obs:  ( 1-x | x) é 1-x dividido por x .
>>
>> --
>> Esta mensagem foi verificada pelo sistema de antivírus e
>> acredita-se estar livre de perigo.
>
>
> --
> Esta mensagem foi verificada pelo sistema de antivírus e
> acredita-se estar livre de perigo.

-- 
Esta mensagem foi verificada pelo sistema de antiv�rus e
 acredita-se estar livre de perigo.



[obm-l] Equação Funcional

2018-06-11 Por tôpico Jeferson Almir
Seja f(x) uma função real definida em R -{0,1} tal que

f(x) + f( 1-x | x ) =1 + x determine f (x) .

Obs:  ( 1-x | x) é 1-x dividido por x .

-- 
Esta mensagem foi verificada pelo sistema de antiv�rus e
 acredita-se estar livre de perigo.



[obm-l] Re: [obm-l] Re: [obm-l] Teorema do Valor Médio

2018-05-23 Por tôpico Jeferson Almir
Tem razão!! Tem que mostrar que a única que satisfaz é a função constante .
Obrigado

Em qua, 23 de mai de 2018 às 17:59, Otávio Araújo <otavio17.ara...@gmail.com>
escreveu:

> Tem que haver uma condição adicional ao enunciado
>
> Em qua, 23 de mai de 2018 17:50, Otávio Araújo <otavio17.ara...@gmail.com>
> escreveu:
>
>> E existe contra exemplo: f constante satisfaz essa condição
>>
>> Em qua, 23 de mai de 2018 17:44, Otávio Araújo <otavio17.ara...@gmail.com>
>> escreveu:
>>
>>> O teorema do valor médio se refere a funções deriváveis. Acho que Vc
>>> está falando do teorema do valor intermediário ou que a função f é derivável
>>>
>>> Em qua, 23 de mai de 2018 17:36, Jeferson Almir <
>>> jefersonram...@gmail.com> escreveu:
>>>
>>>> Como eu uso o teorema do Valor Médio  pra mostrar que não existe função
>>>> real continua tal que f ( x+f(x)) = f(x)?
>>>>
>>>> --
>>>> Esta mensagem foi verificada pelo sistema de antivírus e
>>>> acredita-se estar livre de perigo.
>>>
>>>
> --
> Esta mensagem foi verificada pelo sistema de antivírus e
> acredita-se estar livre de perigo.

-- 
Esta mensagem foi verificada pelo sistema de antiv�rus e
 acredita-se estar livre de perigo.



[obm-l] Teorema do Valor Médio

2018-05-23 Por tôpico Jeferson Almir
Como eu uso o teorema do Valor Médio  pra mostrar que não existe função
real continua tal que f ( x+f(x)) = f(x)?

-- 
Esta mensagem foi verificada pelo sistema de antiv�rus e
 acredita-se estar livre de perigo.



[obm-l] Re: [obm-l] Re: [obm-l] Função Composta

2018-05-13 Por tôpico Jeferson Almir
Boa noite.
Eu só não entendi essa passagem
 “ Para todo a, queremos que N seja igual a 1, 2 ou 5 (os divisores de 50
menores ou iguais a 5).“
Pois pra mim eu teria que levar em conta somente os divisores de 50

Em dom, 13 de mai de 2018 às 19:43, Bruno Visnadi <
brunovisnadida...@gmail.com> escreveu:

> Não sei se ficou meio confuso:
> De fato a função é injetiva, pois se f(a) = f(b) então f^50(a) = f^50(b) e
> a = b. E claramente é sobrejetiva, portanto, é bijetiva. Existem 5! = 120
> bijeções de S em S. Vamos descontar as que não tem a propriedade desejada.
> Em cada bijeção de S em S, dado um a, existe um menor N tal que f^N(a) =
> a. Para todo a, queremos que N seja igual a 1, 2 ou 5 (os divisores de 50
> menores ou iguais a 5).
> Se existem um a cujo N é igual a 3, temos um caso em que f(a) = b, f(b) =
> c e f(c) = a . Existem 10 maneiras de escolher a, b, c de S, duas
> maneiras de escolher o 'ciclo' entre eles (a->b->c ou a->c->b), e mais 2
> maneiras de escolher a imagem dos outros 2 elementos (se forem x e y,
> podemos ter f(x) = x e f(y) = y ou f(x) = y e f(y) = x). Então temos 10*2*2
> = 40 funções deste tipo.
> Se existe um a cujo N é igual a 4, temos um caso em que f(a) = b, f(b) =
> c e f(c) = d e f(d) = a. Temos 5 maneiras de escolher estes 4 elementos de
> S, e mais 6 maneiras de ordenar o 'ciclo' entre eles (basta fixar um deles
> e vemos que são 3! maneiras). Então 6*5 = 30 funções deste tipo.
> Logo a quantidade de funções com as propriedades que buscamos é 120-40-30
> = 50.
>
> Em 13 de maio de 2018 18:03, Jeferson Almir <jefersonram...@gmail.com>
> escreveu:
>
>> Seja S = { 1,2,3,4,5 }, quantas são as funções de f: S -> S tais que
>> f^50(x)= x para todo x pertencente a S ?? ( f^50(x) = fofofo...of(x)
>> Eu provei que ela era injetiva e acho que provei também que ela era
>> sobrejetiva mas minha resposta dar 45 . O gabarito diz que são 50. Desde já
>> agradeço qualquer ajuda.
>>
>> --
>> Esta mensagem foi verificada pelo sistema de antivírus e
>> acredita-se estar livre de perigo.
>
>
> --
> Esta mensagem foi verificada pelo sistema de antivírus e
> acredita-se estar livre de perigo.

-- 
Esta mensagem foi verificada pelo sistema de antiv�rus e
 acredita-se estar livre de perigo.



[obm-l] Re: [obm-l] Re: [obm-l] Re: [obm-l] Re: [obm-l] Função Composta

2018-05-13 Por tôpico Jeferson Almir
Valeu Raph e os demais. Aprendi muito com vcs!!

Em sáb, 12 de mai de 2018 às 20:25, Ralph Teixeira <ralp...@gmail.com>
escreveu:

> Oops, eh verdade, esqueci de mostrar que f nao tem ponto fixo em Z_2005
> (obviamente f nao tem ponto fixo, pois f(f(a))<>a).
>
> Suponha por absurdo que f(a)=a+K.2005 para algum a em {0,1,...2004}, com K
> natural. Entao f(a+K.2005)=f(f(a))=a+2005. Agora, usando nossa
> propriedadezinha:
>
> f(a+K.2005)-f(a)=K.2005
> a+2005 - (a+K.2005) = K.2005
> K = 1/2 (absurdo).
>
> Abraco, Ralph.
>
>
>
> 2018-05-12 2:49 GMT-03:00 Bernardo Freitas Paulo da Costa <
> bernardo...@gmail.com>:
>
>> Oi Ralph,
>>
>> 2018-05-11 20:03 GMT-03:00 Ralph Teixeira <ralp...@gmail.com>:
>> > (Vou supor que 0 eh natural; se nao for, apenas troque 0 por 2005 ali
>> > embaixo e ajeite as coisas)
>> >
>> > Primeiro: f eh injetiva. De fato, f(a)=f(b) => f(f(a))=f(f(b)) =>
>> > a+2005=b+2005 => a=b.
>> >
>> > Segundo: para todo n natural, f(n+2005)=f(f(f(n)))=f(n)+2005. Portanto,
>> por
>> > indução, para qualquer K natural, tem-se
>> > f(n+K.2005)=f(n)+K.2005, ou seja, f(n+K.2005)-f(n)=K.2005.
>> >
>> > VERSÃO CURTA COM TERMINOLOGIA "MOD":
>> > Ou seja, mostramos que   a=b (mod 2005) => f(a)=f(b) (mod 2005).
>> > Agora, se f(m)=n (mod 2005), entao f(n)=f(f(m))=m+2005=m (mod 2005). Ou
>> > seja, f estah bem definida e eh sua propria inversa em Z_2005, o que eh
>> > absurdo, pois Z_2005 tem um numero impar de elementos.
>>
>> Peraí, não entendi direito... se f(n) == n (mod 2005), temos uma
>> função que é sua própria inversa mod 2005.  Temos que excluir este
>> caso...
>>
>> > 2018-05-11 10:42 GMT-03:00 Jeferson Almir <jefersonram...@gmail.com>:
>> >>
>> >> Como provar que nos naturais não existe a função f ( f(n) ) = n + 2005
>> ???
>> >>
>>
>> Abraços,
>> --
>> Bernardo Freitas Paulo da Costa
>>
>> --
>> Esta mensagem foi verificada pelo sistema de antivírus e
>>  acredita-se estar livre de perigo.
>>
>>
>> =
>> Instru�ões para entrar na lista, sair da lista e usar a lista em
>> http://www.mat.puc-rio.br/~obmlistas/obm-l.html
>> =
>>
>
>
> --
> Esta mensagem foi verificada pelo sistema de antivírus e
> acredita-se estar livre de perigo.

-- 
Esta mensagem foi verificada pelo sistema de antiv�rus e
 acredita-se estar livre de perigo.



[obm-l] Função Composta

2018-05-13 Por tôpico Jeferson Almir
Seja S = { 1,2,3,4,5 }, quantas são as funções de f: S -> S tais que
f^50(x)= x para todo x pertencente a S ?? ( f^50(x) = fofofo...of(x)
Eu provei que ela era injetiva e acho que provei também que ela era
sobrejetiva mas minha resposta dar 45 . O gabarito diz que são 50. Desde já
agradeço qualquer ajuda.

-- 
Esta mensagem foi verificada pelo sistema de antiv�rus e
 acredita-se estar livre de perigo.



[obm-l] Função Composta

2018-05-11 Por tôpico Jeferson Almir
Como provar que nos naturais não existe a função f ( f(n) ) = n + 2005 ???

-- 
Esta mensagem foi verificada pelo sistema de antiv�rus e
 acredita-se estar livre de perigo.



[obm-l] Re: [obm-l] Re: [obm-l] Re: Função não periódica

2018-04-15 Por tôpico Jeferson Almir
O Artur já me respondeu algo relacionado .
https://answers.yahoo.com/question/index;_ylt=ArGgI5KmvwfN1NgNFs2qoFPty6IX;_ylv=3?qid=20130107164843AAfIWMj
e em outro email  aqui na lista sobre *g(x) = f(x^a), *

Em 15 de abril de 2018 19:55, Artur Steiner 
escreveu:

> No caso de f(x) = sen(x^2), também podemos fazer assim:
>
> Se f for periódica, então f'(x) = 2x cos(x^2) também é. E como f' é'
> contínua, é limitada. Mas fazendo x_n = raiz(2pi n), n natural, vemos que
> f'(x_n) = raiz(2pi n) vai para oo com n. Temos assim uma contradição que
> mostra que f não é periódica.
>
>
> Artur Costa Steiner
>
> Em Sáb, 14 de abr de 2018 20:16, Artur Costa Steiner <
> artur_stei...@yahoo.com> escreveu:
>
>> Isso é consequência do fato de que x —> sen(x^2) é contínua mas não
>> uniformemente contínua.
>>
>> Artur
>>
>>
>> Enviado do meu iPad
>>
>> Em 14 de abr de 2018, à(s) 1:10 PM, Claudio Buffara <
>> claudio.buff...@gmail.com> escreveu:
>>
>> Que tal começar provando que x --> sen(x^2) não é periódica?
>>
>> 2018-04-14 13:04 GMT-03:00 Claudio Buffara :
>>
>>> Eu também fiquei inseguro em relação a isso e também notei que não
>>> usei (pelo menos não explicitamente) a continuidade de f.
>>>
>>> Mas g(raiz(x+kT)) = g(raiz(x+(k+1)T) não só para um número x fixo,
>>> mas para cada x >= -kT: um intervalo infinito.
>>> Será que isso não é suficiente para estabelecer a periodicidade de g?
>>>
>>> []s,
>>> Claudio.
>>>
>>>
>>> 2018-04-14 11:42 GMT-03:00 Bernardo Freitas Paulo da Costa <
>>> bernardo...@gmail.com>:
>>>
 Oi Claudio,

 2018-04-14 10:54 GMT-03:00 Claudio Buffara :
 > f é periódica (digamos, de período T > 0).
 >
 > Suponhamos que g também seja periódica, digamos de período P.
 >
 > Para todo x, e todo k em N tal que x+kT >= 0, g(raiz(x+kT)) = f(x+kT)
 =
 > f(x+(k+1)T) = g(raiz(x+(k+1)T)) ==>
 > raiz(x+(k+1)T) - raiz(x+kT) = nP, para algum n em N.

 não é verdade que, se g(x) é periódica, e g(x) = g(y), então x - y
 é
 múltiplo do período.  Por exemplo, sin(pi/2 + a) = sin(pi/2 - a),
 para
 todo a.

 > Mas tomando k suficientemente grande, podemos fazer raiz(x+(k+1)T) -
 > raiz(x+kT) tão pequeno quanto quisermos, em particular < P, o que
 contraria
 > raiz(x+(k+1)T) - raiz(x+kT) = nP.

 Intuitivamente, deve mesmo ter a ver com o que você falou sobre o
 limite da diferença das raízes em PA, mas acho que é um pouco mais
 complicado.  Repare que, no enunciado do Arthur, tem um "f
 contínua"...

 > 2018-04-12 15:55 GMT-03:00 Artur Steiner <
 artur.costa.stei...@gmail.com>:
 >>
 >> Suponhamos que f:R —> R seja contínua, periódica e não
 constante. Mostre
 >> que g(x) = f(x^2) não é periódica.
 >>
 >> Artur

 Abraços,
 --
 Bernardo Freitas Paulo da Costa

 --
 Esta mensagem foi verificada pelo sistema de antivírus e
 Â acredita-se estar livre de perigo.


 
 =
 Instru�ões para entrar na lista, sair da lista e usar a lista em
 http://www.mat.puc-rio.br/~obmlistas/obm-l.html
 
 =

>>>
>>>
>>
>> --
>> Esta mensagem foi verificada pelo sistema de antivírus e
>> acredita-se estar livre de perigo.
>>
>>
> --
> Esta mensagem foi verificada pelo sistema de antivírus e
> acredita-se estar livre de perigo.
>

-- 
Esta mensagem foi verificada pelo sistema de antiv�rus e
 acredita-se estar livre de perigo.



[obm-l] Re: [obm-l] Re: [obm-l] Triângulo quase Russo - 12º - 18º - 42º

2018-04-04 Por tôpico Jeferson Almir
Ainda não chegou ... mas se puder mandar pro meu e-mail desde já agradeço
:) .. Abraço Jeferson Almir

Em qua, 4 de abr de 2018 às 10:30, Julio César Saldaña Pumarica <
saldana...@pucp.edu.pe> escreveu:

> Ontem enviei uma solução como arquivo anexo. Era uma foto com a minha
> solução. Parece que o email não chegou, poderia me confirmar?, existe
> alguma restrição quanto anexos?
>
> A resposta é 48, e fiz a solução usando apenas geometria básica.
>
> Obrigado
>
> Julio
> 2018-02-28 7:36 GMT-03:00 Jeferson Almir <jefersonram...@gmail.com>:
>
>> Queria uma ajuda nesse problema de preferência por geometria sintética :)
>>
>> Seja um triângulo ABC isósceles de base BC sendo  A = 12º e os pontos E e
>> D sobre AB e BC respectivamente tal que os ângulos ECB= 42º e DBC =18º.
>> Calcule o ângulo EDB.
>>
> --
>> Esta mensagem foi verificada pelo sistema de antivírus e
>> acredita-se estar livre de perigo.
>
>
> --
> Esta mensagem foi verificada pelo sistema de antivírus e
> acredita-se estar livre de perigo.

-- 
Esta mensagem foi verificada pelo sistema de antiv�rus e
 acredita-se estar livre de perigo.



[obm-l] Revistas Matemáticas

2018-04-02 Por tôpico Jeferson Almir
   Caros amigos assim como muitos desse ambiente sou entusiasta da
matemática e acompanho revistas como a CRUX Mathematicorum,  mas estou
tendo problemas em conseguir a Gazeta Mathematica ( Romenia ) ja tentei
asssinar e não obtive exito. Alguém poderia me ajudar em obter tal
revista?? Abraço Jeferson Almir

-- 
Esta mensagem foi verificada pelo sistema de antiv�rus e
 acredita-se estar livre de perigo.



[obm-l] Re: [obm-l] Re: [obm-l] Re: [obm-l] Triângulo quase Russo - 12º - 18º - 42º

2018-03-01 Por tôpico Jeferson Almir
Opa !! Deu um valor legal. Eu tinha errado a resposta é 48º. Desculpem

Em qui, 1 de mar de 2018 às 11:27, Jeferson Almir <jefersonram...@gmail.com>
escreveu:

> Eu coloquei no Geogebra e deu 48,71º. Deve ter algo errado
>
> Em qua, 28 de fev de 2018 às 21:46, Anderson Torres <
> torres.anderson...@gmail.com> escreveu:
>
>> Em 28 de fevereiro de 2018 11:59, Claudio Buffara
>> <claudio.buff...@gmail.com> escreveu:
>> > Sugestão 1: usando régua e transferidor, desenhe uma figura tão grande e
>> > precisa quanto puder (por exemplo, ocupando a maior parte de uma folha
>> de
>> > A4).
>> > Daí, meça o ângulo EDB com o transferidor e obtenha uma conjectura.
>> > Já será um progresso: ao invés de ter que determinar o valor do ângulo e
>> > provar que seu raciocínio está correto, você precisará apenas provar sua
>> > conjectura.
>> >
>> > Sugestão 2: Como 12 = 360/30, considere um polígono regular convexo de
>> 30
>> > lados inscrito numa circunferência de centro A e tal que B e C sejam
>> > vértices (adjacentes) do polígono.
>>
>> Tô tentando resolver dessa forma, mas acredito ser mais promissor usar
>> um 15-ágono em que
>> os três vértices estão na circunferência circunscrita ao triângulo.
>> Dessa forma é mais fácil ver certas simetrias.
>> Por exemplo, as retas que definem os ângulos inferiores atingem
>> meios-arcos interessantes. Daí fica mais
>> fácil verificar algumas propriedades.
>>
>> > Considere os vértices P e Q do polígono tais que PAB, BAC e CAQ são
>> ângulos
>> > adjacentes (digamos com P, B, C, Q tomados no sentido anti-horário
>> sobre a
>> > circunferência), com PAB = 84 graus e CAQ = 36 graus. Prove que BQ
>> > intersecta AC em D e CP intersecta AB em E.
>> > Será que a reta suporte de DE intersecta a circunferência em pontos que
>> são
>> > vértices do polígono de 30 lados?
>> > Aqui está outra situação em que um desenho bem feito (agora também com
>> um
>> > compasso) pode ajudar.
>> > Ou então, se você usar Geogebra ou algum outro software de geometria
>> > dinâmica...
>> >
>> > []s,
>> > Claudio.
>> >
>> >
>> > 2018-02-28 7:36 GMT-03:00 Jeferson Almir <jefersonram...@gmail.com>:
>> >>
>> >> Queria uma ajuda nesse problema de preferência por geometria sintética
>> :)
>> >>
>> >> Seja um triângulo ABC isósceles de base BC sendo  A = 12º e os pontos
>> E e
>> >> D sobre AB e BC respectivamente tal que os ângulos ECB= 42º e DBC =18º.
>> >> Calcule o ângulo EDB.
>> >>
>> >> --
>> >> Esta mensagem foi verificada pelo sistema de antivírus e
>> >> acredita-se estar livre de perigo.
>> >
>> >
>> >
>> > --
>> > Esta mensagem foi verificada pelo sistema de antivírus e
>> > acredita-se estar livre de perigo.
>>
>> --
>> Esta mensagem foi verificada pelo sistema de antivírus e
>>  acredita-se estar livre de perigo.
>>
>>
>> =
>> Instru�ões para entrar na lista, sair da lista e usar a lista em
>> http://www.mat.puc-rio.br/~obmlistas/obm-l.html
>> =
>>
>

-- 
Esta mensagem foi verificada pelo sistema de antiv�rus e
 acredita-se estar livre de perigo.



[obm-l] Re: [obm-l] Re: [obm-l] Re: [obm-l] Triângulo quase Russo - 12º - 18º - 42º

2018-03-01 Por tôpico Jeferson Almir
Eu coloquei no Geogebra e deu 48,71º. Deve ter algo errado

Em qua, 28 de fev de 2018 às 21:46, Anderson Torres <
torres.anderson...@gmail.com> escreveu:

> Em 28 de fevereiro de 2018 11:59, Claudio Buffara
> <claudio.buff...@gmail.com> escreveu:
> > Sugestão 1: usando régua e transferidor, desenhe uma figura tão grande e
> > precisa quanto puder (por exemplo, ocupando a maior parte de uma folha de
> > A4).
> > Daí, meça o ângulo EDB com o transferidor e obtenha uma conjectura.
> > Já será um progresso: ao invés de ter que determinar o valor do ângulo e
> > provar que seu raciocínio está correto, você precisará apenas provar sua
> > conjectura.
> >
> > Sugestão 2: Como 12 = 360/30, considere um polígono regular convexo de 30
> > lados inscrito numa circunferência de centro A e tal que B e C sejam
> > vértices (adjacentes) do polígono.
>
> Tô tentando resolver dessa forma, mas acredito ser mais promissor usar
> um 15-ágono em que
> os três vértices estão na circunferência circunscrita ao triângulo.
> Dessa forma é mais fácil ver certas simetrias.
> Por exemplo, as retas que definem os ângulos inferiores atingem
> meios-arcos interessantes. Daí fica mais
> fácil verificar algumas propriedades.
>
> > Considere os vértices P e Q do polígono tais que PAB, BAC e CAQ são
> ângulos
> > adjacentes (digamos com P, B, C, Q tomados no sentido anti-horário sobre
> a
> > circunferência), com PAB = 84 graus e CAQ = 36 graus. Prove que BQ
> > intersecta AC em D e CP intersecta AB em E.
> > Será que a reta suporte de DE intersecta a circunferência em pontos que
> são
> > vértices do polígono de 30 lados?
> > Aqui está outra situação em que um desenho bem feito (agora também com um
> > compasso) pode ajudar.
> > Ou então, se você usar Geogebra ou algum outro software de geometria
> > dinâmica...
> >
> > []s,
> > Claudio.
> >
> >
> > 2018-02-28 7:36 GMT-03:00 Jeferson Almir <jefersonram...@gmail.com>:
> >>
> >> Queria uma ajuda nesse problema de preferência por geometria sintética
> :)
> >>
> >> Seja um triângulo ABC isósceles de base BC sendo  A = 12º e os pontos E
> e
> >> D sobre AB e BC respectivamente tal que os ângulos ECB= 42º e DBC =18º.
> >> Calcule o ângulo EDB.
> >>
> >> --
> >> Esta mensagem foi verificada pelo sistema de antivírus e
> >> acredita-se estar livre de perigo.
> >
> >
> >
> > --
> > Esta mensagem foi verificada pelo sistema de antivírus e
> > acredita-se estar livre de perigo.
>
> --
> Esta mensagem foi verificada pelo sistema de antivírus e
>  acredita-se estar livre de perigo.
>
>
> =
> Instru�ões para entrar na lista, sair da lista e usar a lista em
> http://www.mat.puc-rio.br/~obmlistas/obm-l.html
> =
>

-- 
Esta mensagem foi verificada pelo sistema de antiv�rus e
 acredita-se estar livre de perigo.



[obm-l] Triângulo quase Russo - 12º - 18º - 42º

2018-02-28 Por tôpico Jeferson Almir
Queria uma ajuda nesse problema de preferência por geometria sintética :)

Seja um triângulo ABC isósceles de base BC sendo  A = 12º e os pontos E e D
sobre AB e BC respectivamente tal que os ângulos ECB= 42º e DBC =18º.
Calcule o ângulo EDB.

-- 
Esta mensagem foi verificada pelo sistema de antiv�rus e
 acredita-se estar livre de perigo.



[obm-l] Re: [obm-l] Recorrência

2016-10-16 Por tôpico Jeferson Almir
Principles and Techniques in Combinatorics
( Chen chuan-chong ) acredito ser intermediário pra Phoda
Aí desses pesados existe o Introduction to Combinatorics e o
Problems in Combinatorics and Graph Theory ambos do renomado IOAN TOMESCU

Em domingo, 16 de outubro de 2016, Esdras Muniz 
escreveu:

> Olá amigos, gostaria que me passassem eferências de livros ou artigos que
> falem sobre recorrência. Dês de já obrigado.
>
> --
> Esdras Muniz Mota
> Mestrando em Matemática
> Universidade Federal do Ceará
>
>
>
> --
> Esta mensagem foi verificada pelo sistema de antivírus e
> acredita-se estar livre de perigo.

-- 
Esta mensagem foi verificada pelo sistema de antiv�rus e
 acredita-se estar livre de perigo.



[obm-l] Re: [obm-l] Re: [obm-l] Re: [obm-l] [obm-l] Questão Geometria

2016-10-09 Por tôpico Jeferson Almir
Sei que o tópico não tem nada a ver com o problema proposto, mas já postei
2 problemas que não aparecem na caixa da lista e percebi que alguns
receberam pois até responderam. Isso já aconteceu com alguém???

Em 9 de outubro de 2016 15:23, Israel Meireles Chrisostomo <
israelmchrisost...@gmail.com> escreveu:

> Se vc não quiser receber mais emails da obm l envie um emeail para obm l
>
> Em 8 de outubro de 2016 13:15, Matheus Herculano <
> matheusherculan...@gmail.com> escreveu:
>
>> A resposta é para de me mandar isso
>>
>> Em 1 de out de 2016 20:00, "vinicius raimundo" 
>> escreveu:
>>
>>> Será que alguém poria me ajudar na seguinte questão?
>>>
>>>
>>>1.
>>>
>>>(Belarus) Seja O o centro do círculo ex-inscrito do triângulo ABC oposto
>>>ao vértice A. Seja M o ponto médio de AC e seja P a intersec ̧ão das
>>>retas MO e BC. Prove que se ∠BAC = 2∠ACB, então  AB = BP.
>>>
>>>
>>> --
>>> Esta mensagem foi verificada pelo sistema de antivírus e
>>> acredita-se estar livre de perigo.
>>
>>
>> --
>> Esta mensagem foi verificada pelo sistema de antivírus e
>> acredita-se estar livre de perigo.
>>
>
>
> --
> Esta mensagem foi verificada pelo sistema de antivírus e
> acredita-se estar livre de perigo.
>

-- 
Esta mensagem foi verificada pelo sistema de antiv�rus e
 acredita-se estar livre de perigo.



[obm-l] Re: [obm-l] Re: [obm-l] Re: [obm-l] [obm-l] Questão Geometria

2016-10-09 Por tôpico Jeferson Almir
Perdão foi processado sim na Mail Archive acabo de constatar mas demorou
alguns dias para aparecer. Valeu!!

Em 9 de outubro de 2016 17:40, Jeferson Almir <jefersonram...@gmail.com>
escreveu:

> Sei que o tópico não tem nada a ver com o problema proposto, mas já postei
> 2 problemas que não aparecem na caixa da lista e percebi que alguns
> receberam pois até responderam. Isso já aconteceu com alguém???
>
> Em 9 de outubro de 2016 15:23, Israel Meireles Chrisostomo <
> israelmchrisost...@gmail.com> escreveu:
>
>> Se vc não quiser receber mais emails da obm l envie um emeail para obm l
>>
>> Em 8 de outubro de 2016 13:15, Matheus Herculano <
>> matheusherculan...@gmail.com> escreveu:
>>
>>> A resposta é para de me mandar isso
>>>
>>> Em 1 de out de 2016 20:00, "vinicius raimundo" <vini.raimu...@gmail.com>
>>> escreveu:
>>>
>>>> Será que alguém poria me ajudar na seguinte questão?
>>>>
>>>>
>>>>1.
>>>>
>>>>(Belarus) Seja O o centro do círculo ex-inscrito do triângulo ABC oposto
>>>>ao vértice A. Seja M o ponto médio de AC e seja P a intersec ̧ão
>>>>das retas MO e BC. Prove que se ∠BAC = 2∠ACB, então  AB = BP.
>>>>
>>>>
>>>> --
>>>> Esta mensagem foi verificada pelo sistema de antivírus e
>>>> acredita-se estar livre de perigo.
>>>
>>>
>>> --
>>> Esta mensagem foi verificada pelo sistema de antivírus e
>>> acredita-se estar livre de perigo.
>>>
>>
>>
>> --
>> Esta mensagem foi verificada pelo sistema de antivírus e
>> acredita-se estar livre de perigo.
>>
>
>

-- 
Esta mensagem foi verificada pelo sistema de antiv�rus e
 acredita-se estar livre de perigo.



Re: [obm-l] Re: Geometria

2016-09-11 Por tôpico Jeferson Almir
Obrigado a todos que responderam, eu mandei varias vezes o email pra lista
pois eu achava que ele não estava sendo entregue pois na timeline da lista
ele não aparece. Desde já obrigado

Em 10 de setembro de 2016 22:42, Carlos Victor <victorcar...@globo.com>
escreveu:

>
>
>
> Oi Jeferson,
>
> Tome E sobre BD tal que o ângulo EAB seja 30º. Observe que o ângulo ADB é
> igual a 100º e que o ângulo DAE é igual a 20º. Daí o ângulo AED é igual a
> 60º. Como E está na bissetriz de ACB, então o ângulo AEC é igual a 120º.
> Observe agora que D é o ponto de encontro das bissetrizes internas do
> triângulo AEC e consequentemente o ângulo BDC é igual a 110º.
>
> Abraços
>
>
>
> Carlos  Victor
>
> Em 10/09/2016 17:34, Jeferson Almir escreveu:
>
> Olá pessoa queria uma ajuda nessa questão
>
> A figura em anexo mostra um triângulo *ABC*. *D* é um ponto interior onde
> a medida dos ângulos *CAD*, *ABD*, *CBD*, e *BAD* são 20º, 30º, 40º e 50º
> , respectivamente. Encontre a medida do ângulo *BDC*.
>
> Em 28 de agosto de 2016 18:31, Jeferson Almir <jefersonram...@gmail.com>
> escreveu:
>
>> Olá pessoa queria uma ajuda nessa questão
>>
>> A figura em anexo mostra um triângulo *ABC*. *D* é um ponto interior
>> onde a medida dos ângulos *CAD*, *ABD*, *CBD*, e *BAD* são 20º, 30º, 40º
>> e 50º , respectivamente. Encontre a medida do ângulo *BDC*.
>>
>>
>>
> --
> Esta mensagem foi verificada pelo sistema de antivrus e
> acredita-se estar livre de perigo.
>
>
> --
> Esta mensagem foi verificada pelo sistema de antivírus e
> acredita-se estar livre de perigo.
>

-- 
Esta mensagem foi verificada pelo sistema de antiv�rus e
 acredita-se estar livre de perigo.



[obm-l] Re: Geometria

2016-09-10 Por tôpico Jeferson Almir
Olá pessoa queria uma ajuda nessa questão

A figura em anexo mostra um triângulo *ABC*. *D* é um ponto interior onde a
medida dos ângulos *CAD*, *ABD*, *CBD*, e *BAD* são 20º, 30º, 40º e 50º ,
respectivamente. Encontre a medida do ângulo *BDC*.

Em 28 de agosto de 2016 18:31, Jeferson Almir <jefersonram...@gmail.com>
escreveu:

> Olá pessoa queria uma ajuda nessa questão
>
> A figura em anexo mostra um triângulo *ABC*. *D* é um ponto interior onde
> a medida dos ângulos *CAD*, *ABD*, *CBD*, e *BAD* são 20º, 30º, 40º e 50º
> , respectivamente. Encontre a medida do ângulo *BDC*.
>
>
>

-- 
Esta mensagem foi verificada pelo sistema de antiv�rus e
 acredita-se estar livre de perigo.



[obm-l] Re: [obm-l] Dúvida sobre a Obm U

2016-07-25 Por tôpico Jeferson Almir
Boa iniciativa Sandino!!
Um prova que se aproxima bastante é a do Putnam ( universitária americana )
e o livro Putnam and Beyond seria um bom começo.
Refazer prova passadas da OBMU  e depois ver a solução  possíveis dúvidas é
um ótimo começo. Existe também a universitária colombiana que que a parte
de álgebra linear eu particularmente acho ótimo. Abraço

Em segunda-feira, 25 de julho de 2016, Tiago Sandino <
tiagosandi...@gmail.com> escreveu:

> Oi pessoal.
> Tem diversos livros de olimpíadas para graduandos (undergrads) ou com
> capítulos de temas exclusivamente (até onde eu saiba) universitários.
> Grátis na net, que eu saiba, tem muita coisa no AOPS. Dois links aqui:
> 1) *Fórum*: https://www.artofproblemsolving.com/community/c7_college_math
> 2) *Fórum por Competições*:
> https://www.artofproblemsolving.com/community/c15_undergraduate_contests
>
> Sou do Ceará, tava meio afastado da Matemática, mas fiz as pazes com ela
> recentemente. Se alguém quiser formar um grupo de estudo pelo Whatsapp...
> segue meu número: 85 9 9913 4896.
>
> Att.
> Tiago Sandino
>
> Em 25 de julho de 2016 10:20, Raul Alves  > escreveu:
>
>> Também tenho interesse na OBMU, e a 1ª fase tá chegando.
>> Se algum professor puder organizar algum material de apoio, seria de
>> grande ajuda
>>
>> Em 25 de julho de 2016 10:09, Otávio Araújo > > escreveu:
>>
>>>
>>>
>>> Pois é, se algum professor com experiência em olimpíadas, como o Nicolau
>>> por exemplo, respondesse minha pergunta seria de grande ajuda
>>>
>>> Em 24 de jul de 2016, às 23:25, Israel Meireles Chrisostomo <
>>> israelmchrisost...@gmail.com
>>> >
>>> escreveu:
>>>
>>> Boa pergunta, eu também tenho interesse em participar da OBM U e
>>> gostaria de umas dicas
>>>
>>> Em 16 de julho de 2016 13:29, Otávio Araújo >> > escreveu:
>>>
 Galera, gostaria que vocês me dessem dicas de o que estudar, como
 estudar e por quais livros e materiais estudar para a prova da Obm nível
 universitário...
 Estou muito interessado em participar, mas fico meio confuso por onde
 estudar...
 Por favor me ajudem
 --
 Esta mensagem foi verificada pelo sistema de antivírus e
 Â acredita-se estar livre de perigo.



 =
 Instruções para entrar na lista, sair da lista e usar a lista em
 http://www.mat.puc-rio.br/~obmlistas/obm-l.html

 =

>>>
>>>
>>> --
>>> Esta mensagem foi verificada pelo sistema de antivírus e
>>> acredita-se estar livre de perigo.
>>>
>>>
>>> --
>>> Esta mensagem foi verificada pelo sistema de antivírus e
>>> acredita-se estar livre de perigo.
>>>
>>
>>
>>
>> --
>> *Raul Lima Alves*
>>
>> *Estagiário na Aton Engenharia*
>> *Estudante de Engenharia de Computação - UFBA*
>> *Telefone: (71) 9103-0878*
>> *Facebook: *https://www.facebook.com/raul.alves.161
>> *LinkedIn*: https://br.linkedin.com/in/raul-alves-8b090228
>> 
>>
>>
>>
>> --
>> Esta mensagem foi verificada pelo sistema de antivírus e
>> acredita-se estar livre de perigo.
>>
>
>
> --
> Esta mensagem foi verificada pelo sistema de antivírus e
> acredita-se estar livre de perigo.

-- 
Esta mensagem foi verificada pelo sistema de antiv�rus e
 acredita-se estar livre de perigo.



[obm-l] Progressão Aritmetica

2016-05-29 Por tôpico Jeferson Almir
Desde já agradeço qualquer idéia ou ajuda

Seja [image: $f: \mathbb{N} \rightarrow \mathbb{N}$] uma função Injetiva

a) Mostre que existe uma progressão aritmética de três termos  [image:
$a$], [image:
$a+d$], [image: $a+2d$] tal que:

[image: $f(a)

[obm-l] Número de Elementos

2016-03-02 Por tôpico Jeferson Almir
Caros peço ajuda nesse problema

Ache todos os conjuntos [image: $A,B,C,D$] com números iguais de elementos
tais que:



(A \ B) ∩ C =D



(B \ C) ∩ D =A



(C \ D) ∩ A =B



(D \ A) ∩ B =C

-- 
Esta mensagem foi verificada pelo sistema de antiv�rus e
 acredita-se estar livre de perigo.



[obm-l] Re: [obm-l] Re: [obm-l] Função Sobrejetiva

2016-02-03 Por tôpico Jeferson Almir
Peço ajuda na seguinte questão:

Seja f: R -> Z tal que f(x) = [ x ∙ {x} ]

a) Mostre que f(x) é sobrejetiva

b) Resolva a equação [ x ∙ {x} ]= [ x ∙ [x] ]
onde [ x ] é a parte inteira e { x } é a parte fracionária

Em 17 de setembro de 2015 13:04, Esdras Muniz <esdrasmunizm...@gmail.com>
escreveu:

> Cara, vc pode fazer isso, pega duas sequências x_n e y_n, com
> lim f(x_n)=+infinito elim f(y_n)=-infinito, e lim(x_n)=+infinito e
> lim(y_n)=-infinito.
> Daí tu usa que f é contínua.
> vc pode pegar x_n=2kpi+pi/2 e y_n=-2kpi-pi/2.
>
> Em 17 de setembro de 2015 12:27, Jeferson Almir <jefersonram...@gmail.com>
> escreveu:
>
>> 1. Provar que a função f( x ) = (x^3)sen( x ) é Sobrejetiva.
>>
>> A ideia que penso e que peço ajuda é que todo x real pode ser
>> representado da forma x = 2kpi + 2/pi isso é válido ??? Caso seja, o
>> problema está resolvido!!!
>>
>>
>> --
>> Esta mensagem foi verificada pelo sistema de antivírus e
>> acredita-se estar livre de perigo.
>
>
>
>
> --
> Esdras Muniz Mota
> Mestrando em Matemática
> Universidade Federal do Ceará
>
>
>
> --
> Esta mensagem foi verificada pelo sistema de antivírus e
> acredita-se estar livre de perigo.


Re: [obm-l] Livros

2016-01-14 Por tôpico Jeferson Almir
Reintero o meu interesse por esses livros, caso alguém já obteve poderia
disponibilizar uma pasta compartilhada no Dropbox seria uma boa ideia.
Abraço Jeferson Almir

Em quinta-feira, 14 de janeiro de 2016, Giovanni Celestre <
ggabrie...@gmail.com> escreveu:

> Eu também, Por favor
> Obrigado
>
> 2016-01-14 13:01 GMT-02:00 Vanderlei Nemitz <vanderma...@gmail.com
> <javascript:_e(%7B%7D,'cvml','vanderma...@gmail.com');>>:
>
>> Eu quero, Israel!
>>
>> Obrigado!
>>
>> Vanderlei
>>
>> Em 14 de janeiro de 2016 12:54, Israel Meireles Chrisostomo <
>> israelmchrisost...@gmail.com
>> <javascript:_e(%7B%7D,'cvml','israelmchrisost...@gmail.com');>> escreveu:
>>
>>> *PROBLEMAS DE ALTA DIFICULDAD - 300 Problemas Resolvidos*
>>>
>>> *Métodos Alternativos para a Resolução de Equações e Inequações - 350
>>> Problemas Resolvidos*Eu tenho esses doi em espanhol, não é pdf, é o
>>> livro mesmo!Se tiver interessado mande um email para mim!
>>>
>>> Em 14 de janeiro de 2016 11:07, benedito freire <bened...@ufrnet.br
>>> <javascript:_e(%7B%7D,'cvml','bened...@ufrnet.br');>> escreveu:
>>>
>>>> Por favor, escreva o nome do autor completo.Talvez
>>>> eu possa conseguir
>>>> --
>>>> De: Vanderlei Nemitz
>>>> <javascript:_e(%7B%7D,'cvml','vanderma...@gmail.com');>
>>>> Enviada em: ‎14/‎01/‎2016 11:12
>>>> Para: OBM <javascript:_e(%7B%7D,'cvml','obm-l@mat.puc-rio.br');>
>>>> Assunto: Re: [obm-l] Livros
>>>>
>>>> Você tem algum deles, Regis? Eu tinha o PDF de dois deles, em Russo,
>>>> mas o pendrive estragou e perdi :(
>>>>
>>>> Em 14 de janeiro de 2016 11:01, Jefferson Cândido <
>>>> jjjeffer...@gmail.com
>>>> <javascript:_e(%7B%7D,'cvml','jjjeffer...@gmail.com');>> escreveu:
>>>>
>>>>> Muito bom! Se puder mandar também para meu e-mail,
>>>>> jjjeffer...@gmail.com
>>>>> <javascript:_e(%7B%7D,'cvml','jjjeffer...@gmail.com');>, agradeço!
>>>>>
>>>>> Em 13 de janeiro de 2016 21:45, Vanderlei Nemitz <
>>>>> vanderma...@gmail.com
>>>>> <javascript:_e(%7B%7D,'cvml','vanderma...@gmail.com');>> escreveu:
>>>>>
>>>>>> *PROBLEMAS DE ALTA DIFICULDAD - 300 Problemas Resolvidos*
>>>>>> *Métodos de Resoluções e Demonstrações de Desigualdades - ** 367
>>>>>> Problemas*
>>>>>> *Métodos Alternativos para a Resolução de Equações e Inequações - 350
>>>>>> Problemas Resolvidos*
>>>>>>
>>>>>> *Qualquer um desses já seria uma grande ajuda!*
>>>>>>
>>>>>> *Obrigado!*
>>>>>>
>>>>>> Em 13 de janeiro de 2016 21:33, regis barros <
>>>>>> regisgbar...@yahoo.com.br
>>>>>> <javascript:_e(%7B%7D,'cvml','regisgbar...@yahoo.com.br');>>
>>>>>> escreveu:
>>>>>>
>>>>>>> Olá Vanderlei
>>>>>>> Quais livros do suprun você precisa?
>>>>>>>
>>>>>>> Regis
>>>>>>>
>>>>>>>
>>>>>>> Em Quarta-feira, 13 de Janeiro de 2016 14:35, Vanderlei Nemitz <
>>>>>>> vanderma...@gmail.com
>>>>>>> <javascript:_e(%7B%7D,'cvml','vanderma...@gmail.com');>> escreveu:
>>>>>>>
>>>>>>>
>>>>>>> Boa tarde! Alguém tem os PDFs dos livros do Suprún? Pode ser até em
>>>>>>> russo mesmo! Ou mesmo tenha e queira vender os livros físicos? Preciso
>>>>>>> muito deles, mas está em falta.
>>>>>>>
>>>>>>> Obrigado!
>>>>>>>
>>>>>>>
>>>>>>>
>>>>>>
>>>>>
>>>>>
>>>>> --
>>>>> É preciso amar as pessoas como se não houvesse amanhã...
>>>>>
>>>>> Jefferson Cândido -
>>>>>
>>>>
>>>>
>>>
>>
>


[obm-l] Equação da Cônica

2015-11-19 Por tôpico Jeferson Almir
Qual o valor de *a* na equação da cônica xˆ2 -3xy+ *a*yˆ2 + 3x -5y +2 =0
para que a cônica represente  um par de retas???

Eu montei uma equação do segundo grau em x e forçando o delta igual a zero
e cheguei na resposta a = 2 que é o que o gabarito afirma mas não entendi.
Alguém poderia resolver de outra maneira ou explicar?? Desde já Obrigado

-- 
Esta mensagem foi verificada pelo sistema de antiv�rus e
 acredita-se estar livre de perigo.



[obm-l] Função Sobrejetiva

2015-09-17 Por tôpico Jeferson Almir
1. Provar que a função f( x ) = (x^3)sen( x ) é Sobrejetiva.

A ideia que penso e que peço ajuda é que todo x real pode ser representado
da forma x = 2kpi + 2/pi isso é válido ??? Caso seja, o problema está
resolvido!!!

-- 
Esta mensagem foi verificada pelo sistema de antiv�rus e
 acredita-se estar livre de perigo.



[obm-l] Re: [obm-l] transcedência

2015-07-07 Por tôpico Jeferson Almir
Também fiquei curioso e reforço à pergunta do Israel Terence! Como provar
que todo número construtivel com régua e compasso è raiz de um polinômio de
coeficientes inteiro?

Em domingo, 5 de julho de 2015, Israel Meireles Chrisostomo 
israelmchrisost...@gmail.com escreveu:

 E como se prova que todo número construtível com régua e compasso é raiz
 de um polinômio de coeficientes inteiros?Vc teria algum material para me
 indicar?

 Em 5 de julho de 2015 19:29, terence thirteen peterdirich...@gmail.com
 javascript:_e(%7B%7D,'cvml','peterdirich...@gmail.com'); escreveu:


 Em 3 de julho de 2015 19:34, Israel Meireles Chrisostomo 
 israelmchrisost...@gmail.com
 javascript:_e(%7B%7D,'cvml','israelmchrisost...@gmail.com'); escreveu:

 Olá pessoal, o fato de pi ser transcendente implica que não existe um
 segmento de reta de tamanho pi?Estava pensando nisso pq li que a quadratura
 do círculo é impossível por causa da transcendência de pi...



 É claro que não.
 A quadratura do círculo trata-se, basicamente, de construir com régua e
 compasso um quadrado com a mesma área de um círculo de raio conhecido.

 Isto é impossível, porque todo número construtível com régua e compasso é
 raiz de um polinômio de coeficientes inteiros, enquanto pi não é.






 --
 Esta mensagem foi verificada pelo sistema de antivírus e
 acredita-se estar livre de perigo.




 --
 /**/
 神が祝福

 Torres

 --
 Esta mensagem foi verificada pelo sistema de antivírus e
 acredita-se estar livre de perigo.



 --
 Esta mensagem foi verificada pelo sistema de antivírus e
 acredita-se estar livre de perigo.

-- 
Esta mensagem foi verificada pelo sistema de antiv�rus e
 acredita-se estar livre de perigo.



[obm-l] Ternas Pitagóricas

2015-05-18 Por tôpico Jeferson Almir
Peço ajuda nas seguintes questões

1) determine todos x,y,z inteiros tais que x^2 + 2y^2 = z^2  onde mdc(
x,y,z)=1

2) Determine todos inteiros x^2 + y^2 = 1997( x- y )

-- 
Esta mensagem foi verificada pelo sistema de antiv�rus e
 acredita-se estar livre de perigo.



Re: [obm-l] Gavetas

2015-05-10 Por tôpico Jeferson Almir
Além disso, além de provar que existe 2  inteiros que diferem 9 podemos
provar que existem 2 inteiros que diferem 10 ou 12 ou 13 mas
surpreendentemente, não existe necessariamente inteiros que diferem 11.

Em domingo, 10 de maio de 2015, Ralph Teixeira ralp...@gmail.com escreveu:

 Vamos repartir A em 9 conjuntos:

 A1={1,10,19,28,...,91,100}
 A2={2,11,20,29,...,92}
 A3={3,12,21,30,...,93}
 ...
 A9=(9,18,27,36,...,99}

 Como sao 55 numeros escolhidos e 9 conjuntos, pelo menos um conjunto tem
 pelo menos [55/9]+1=7 numeros escolhidos.

 (Se cada um tivesse 6 ou menos, teriamos um total menor ou igual a 6*9=54
 escolhas.)

 Mas cada conjunto tem 11 elementos (bom, ok, A1 tem 12). Se voce escolher
 7 elementos dentre uma fila de 11 (ou 12), voce vai ter que escolher dois
 adjacentes!

 (Se esta parte final necessita clarificacao: no conjunto de 12 elementos
 A1={1,10,,...,100}, considere os 6 pares {1,10},{19,28},...{90,99}.
 Escolhendo 7 elementos, voce vai ter que escolher 2 num mesmo par.
 Raciocinio analogo vale nos outros Ai que tem menos elementos, basta deixar
 o ultimo par com um elemento soh).

 Abraco, Ralph.

 P.S.: Alias, acabo de ver que tem uma solucao mais direta. Basta olhar
 para os conjuntos
 (01,10),(02,11),(03,12),(04,13),(05,14),(06,15),(07,16),(08,17),(09,18)

 (19,28),(20,29),(21,30),(22,31),(23,32),.(27,36)

 (37,46),(38,47),..(45,54)

 (55,64),.(63,72)

 (73,82),.(81,90)
 (91,100),(92),(93),(94),(95),(96),(97),(98),(99)
 Ah, droga, coloquei parenteses ao inves de chaves, mas ok. Bom, conte: sao
 9*5 nas 5 primeiras linhas, mais 9 conjuntos na ultima, ou seja, um total
 de 54 conjuntos. Como sao 55 numeros escolhidos, tem que ter 2 em algum
 mesmo conjunto, acabou.

 2015-05-10 15:37 GMT-03:00 marcone augusto araújo borges 
 marconeborge...@hotmail.com
 javascript:_e(%7B%7D,'cvml','marconeborge...@hotmail.com');:

 Do conjunto A = {1,2,...100} escolhemos 55 números.Mostrar que entre os
 números escolhidos
 existem 2 cuja diferença é 9

 --
 Esta mensagem foi verificada pelo sistema de antivírus e
 acredita-se estar livre de perigo.



 --
 Esta mensagem foi verificada pelo sistema de antivírus e
 acredita-se estar livre de perigo.

-- 
Esta mensagem foi verificada pelo sistema de antiv�rus e
 acredita-se estar livre de perigo.



[obm-l] Re: [obm-l] Como faz exercícios desse tipo?

2015-05-10 Por tôpico Jeferson Almir
OBs: w^k= cis(2kPi/6)

Em domingo, 10 de maio de 2015, Jeferson Almir jefersonram...@gmail.com
escreveu:

 Raízes da unidade!! ... Pelo algoritmo da divisão temos g(x^12) = g(x)q(x)
 + r(x) , onde grau(r(x)) 5 agora vc analisa as raízes da unidade de x^6=1
 : que serão w^k=1 onde k=0,1,2,3,4,5 e monta o sistema sobre r(x) aplicando
 o valor dessas raízes pois r(x) = ax^4 + bx^3 + cx^2 + dx + e  elas irão
 zerar g(x) agora é resolver o sistema utilizando as propriedades das raízes
 da unidade.

 Em domingo, 10 de maio de 2015, Israel Meireles Chrisostomo 
 israelmchrisost...@gmail.com
 javascript:_e(%7B%7D,'cvml','israelmchrisost...@gmail.com'); escreveu:

 Talvez vc poderia observar que -1 é raiz do polinômio,daí vc pode
  fatorar o polinômio como (x-(-1))Q(x) e talvez procurar outras raízes, pq
 aí vc pode fazer a divisão por binômios do tipo (x+1) pois assim  vc
 resolve facilmente pelo algoritmo de briott ruffini, conhece?

 Em 9 de maio de 2015 18:42, Gabriel Tostes gtos...@icloud.com escreveu:

 (EUA/83) Sabendo que g(x) = x^5 + x^4 + x^3 + x^2 + x + 1. Calcule o
 resto da divisão entre polinômios g(x^12) e g(x)

 Dado f(x) =  x^4 + x^3 + x^2 + x + 1, o resto da divisão de f(x^5) por
 f(x) é:

 --
 Esta mensagem foi verificada pelo sistema de antivírus e
 acredita-se estar livre de perigo.



 --
 Esta mensagem foi verificada pelo sistema de antivírus e
 acredita-se estar livre de perigo.



-- 
Esta mensagem foi verificada pelo sistema de antiv�rus e
 acredita-se estar livre de perigo.



[obm-l] Re: [obm-l] Como faz exercícios desse tipo?

2015-05-10 Por tôpico Jeferson Almir
Raízes da unidade!! ... Pelo algoritmo da divisão temos g(x^12) = g(x)q(x)
+ r(x) , onde grau(r(x)) 5 agora vc analisa as raízes da unidade de x^6=1
: que serão w^k=1 onde k=0,1,2,3,4,5 e monta o sistema sobre r(x) aplicando
o valor dessas raízes pois r(x) = ax^4 + bx^3 + cx^2 + dx + e  elas irão
zerar g(x) agora é resolver o sistema utilizando as propriedades das raízes
da unidade.

Em domingo, 10 de maio de 2015, Israel Meireles Chrisostomo 
israelmchrisost...@gmail.com escreveu:

 Talvez vc poderia observar que -1 é raiz do polinômio,daí vc pode  fatorar
 o polinômio como (x-(-1))Q(x) e talvez procurar outras raízes, pq aí vc
 pode fazer a divisão por binômios do tipo (x+1) pois assim  vc resolve
 facilmente pelo algoritmo de briott ruffini, conhece?

 Em 9 de maio de 2015 18:42, Gabriel Tostes gtos...@icloud.com
 javascript:_e(%7B%7D,'cvml','gtos...@icloud.com'); escreveu:

 (EUA/83) Sabendo que g(x) = x^5 + x^4 + x^3 + x^2 + x + 1. Calcule o
 resto da divisão entre polinômios g(x^12) e g(x)

 Dado f(x) =  x^4 + x^3 + x^2 + x + 1, o resto da divisão de f(x^5) por
 f(x) é:

 --
 Esta mensagem foi verificada pelo sistema de antivírus e
 acredita-se estar livre de perigo.



 --
 Esta mensagem foi verificada pelo sistema de antivírus e
 acredita-se estar livre de perigo.

-- 
Esta mensagem foi verificada pelo sistema de antiv�rus e
 acredita-se estar livre de perigo.



Re: [obm-l] N pontos

2015-05-06 Por tôpico Jeferson Almir
Olhe na timeline da lista que esse problema acabou de ser respondido
elegantemente pelo Ralph.

Em quarta-feira, 6 de maio de 2015, Mariana Groff 
bigolingroff.mari...@gmail.com escreveu:

 Boa noite,
 Estou com dúvida no seguinte problema, alguém poderia ajudar-me?

 Dados n pontos em uma circunferência se escreve ao lado de um deles um 1 e
 ao lado de cada um dos outros um 0. A operação permitida consiste em
 escolher um ponto que tenha um 1 e trocar o número desse ponto e também os
 números dos seus dois vizinhos, o da esquerda e o da direita (onde há 1 se
 escreve 0 e onde há 0 se escreve 1).
  a) Se n = 101, mostre que se pode conseguir, mediante uma sucessão de
 operações permitidas, que cada um dos n pontos tenha escrito 0.
  b) Se n = 102, mostre que é impossível obter todos 0.

 Obrigada,
 Mariana

 --
 Esta mensagem foi verificada pelo sistema de antivírus e
 acredita-se estar livre de perigo.

-- 
Esta mensagem foi verificada pelo sistema de antiv�rus e
 acredita-se estar livre de perigo.



[obm-l] Colorir Grafos

2015-05-02 Por tôpico Jeferson Almir
Dado um grafo com N vértices

1° jogador = vai colocando as arestas

2° jogador = vai pintando as arestas com as cores  A ou V

O Jogo acaba quando formar um triângulo monocromático.
Por quanto tempo(número de jogadas) o 2° jogador pode sobreviver??

E se for 3 cores??

-- 
Esta mensagem foi verificada pelo sistema de antiv�rus e
 acredita-se estar livre de perigo.



Re: [obm-l] Provar que...

2014-12-20 Por tôpico Jeferson Almir
Use médias ... M.A  M.G
Algo assim (1+ 2 + 3+...+100)/100 = (1.2.3 ..100)^1/100
Do lado esquerdo vc usa soma de gauss ai fica (50.101)/100  (100!)^1/100
 vou ver se faço as conta aqui mais detalhado e mando...


Em sábado, 20 de dezembro de 2014, Bernardo Freitas Paulo da Costa 
bernardo...@gmail.com escreveu:

 2014-12-20 0:22 GMT-02:00 Maikel Andril Marcelino 
 maikinho0...@hotmail.com javascript:;:
  Mas 50x51  50², temos um problema!

 49*52  50*50 também. Talvez seja melhor cancelar o 50 que aparece dos
 dois lados, daí fica 49*51, 48*52, etc, que são (a-b)*(a+b)  a*a. Mas
 daí vai sobrar o 100. Falta pouco.

  From: dr.dhe...@outlook.com javascript:;
  To: obm-l@mat.puc-rio.br javascript:;
  Subject: RE: [obm-l] Provar que...
  Date: Sat, 20 Dec 2014 05:14:46 +0300
 
 
  Tenta reagrupar 100!, talvez algo como (1*100)(2*99)(3*98)...(50*51), dai
  você terá 50 produtos, cada um deles é equiparável a 50² (a saber menor),
  dai tem que argumentar um pouquinho, mas acho que sai.
 
  Abraços
  Edu
 
  
  From: maikinho0...@hotmail.com javascript:;
  To: obm-l@mat.puc-rio.br javascript:;
  Subject: [obm-l] Provar que...
  Date: Sat, 20 Dec 2014 04:44:26 +0300
 
  100!  50^100, não estou conseguindo galera. Um abraço Carlos Gomes.

 --
 Bernardo Freitas Paulo da Costa

 --
 Esta mensagem foi verificada pelo sistema de antivírus e
  acredita-se estar livre de perigo.


 =
 Instru�ões para entrar na lista, sair da lista e usar a lista em
 http://www.mat.puc-rio.br/~obmlistas/obm-l.html
 =


-- 
Esta mensagem foi verificada pelo sistema de antiv�rus e
 acredita-se estar livre de perigo.



Re: [obm-l] Equacao funcional.

2014-08-26 Por tôpico Jeferson Almir
Aproveitando o momento alguém poderia me ajudar nessa questão??

Determine todas as funções contínuas que projeta três termos sucessivos de
uma progressão aritmética em três termos de uma progressão geométrica.
Desde já agradeço qualquer ajuda.


Em 26 de agosto de 2014 07:40, Douglas Oliveira de Lima 
profdouglaso.del...@gmail.com escreveu:

 Espetaculo, muito obrigado!!


 Em 26 de agosto de 2014 05:26, g...@impa.br escreveu:

Caro Douglas,
Fazendo y=f(x): f(x^2+f(x))+f(0)=2f(f(x))+2f(x)^2.
Fazendo y=-x^2: f(0)+f(f(x)+x^2)=2f(f(x))+2x^4.
Comparando, temos f(x)^2=x^4, donde, para todo x, f(x)=x^2 ou
 f(x)=-x^2. Em particular, f(0)=0. Fazendo então x=0 temos f(y)+f(-y)=2y^2,
 mas f(y) e f(-y) pertencem a {y^2, -y^2}, donde necessariamente
 f(y)=f(-y)=y^2. Assim, f(x)=x^2 para todo x real.
Abraços,
  Gugu


 Quoting Douglas Oliveira de Lima profdouglaso.del...@gmail.com:

  Caos amigos preciso de uma ajuda na seguinte questão, desde ja agradeço!!

 Problema:  Se f(x^2+y)+f(f(x)-y)=2f(f(x))+2y^2 para todos x,t
 pertencentes
 aos reais, determinar todas as funções f:R-R.

 Douglas Oliveira.

 --
 Esta mensagem foi verificada pelo sistema de antiv�rus e
  acredita-se estar livre de perigo.





 
 This message was sent using IMP, the Internet Messaging Program.



 --
 Esta mensagem foi verificada pelo sistema de antivírus e
 acredita-se estar livre de perigo.


 =
 Instru�ões para entrar na lista, sair da lista e usar a lista em
 http://www.mat.puc-rio.br/~obmlistas/obm-l.html
 =



 --
 Esta mensagem foi verificada pelo sistema de antivírus e
 acredita-se estar livre de perigo.


-- 
Esta mensagem foi verificada pelo sistema de antiv�rus e
 acredita-se estar livre de perigo.



[obm-l] Re: Princípio da indução finita

2014-07-24 Por tôpico Jeferson Almir
Como provar isso cassio???

Em quinta-feira, 24 de julho de 2014, Cassio Anderson Feitosa 
cassiofeito...@gmail.com escreveu:

 O P.B. O, e as duas formas de indução são equivalentes entre si.


 Em 23 de julho de 2014 13:16, Jeferson Almir jefersonram...@gmail.com
 javascript:_e(%7B%7D,'cvml','jefersonram...@gmail.com'); escreveu:

 Caros amigos o P.B.O  princípio da boa ordenação é consequência do
 princípio da indução finita ou eles são equivalentes ?? Desde agradeço o
 esclarecimento ou uma possível prova.
 --
 Esta mensagem foi verificada pelo sistema de antivírus e
 acredita-se estar livre de perigo.




 --
 Cássio Anderson
 Graduando em Matemática - UFPB

 --
 Esta mensagem foi verificada pelo sistema de antivírus e
 acredita-se estar livre de perigo.

-- 
Esta mensagem foi verificada pelo sistema de antiv�rus e
 acredita-se estar livre de perigo.



[obm-l] Princípio da indução finita

2014-07-23 Por tôpico Jeferson Almir
Caros amigos o P.B.O  princípio da boa ordenação é consequência do
princípio da indução finita ou eles são equivalentes ?? Desde agradeço o
esclarecimento ou uma possível prova.

-- 
Esta mensagem foi verificada pelo sistema de antiv�rus e
 acredita-se estar livre de perigo.



[obm-l] Re: [obm-l] compilação em um só PDF de milhares, muito mais de 5000

2014-05-28 Por tôpico Jeferson Almir
Alguém conseguiu baixar no Scribd?? Não estou conseguindo caso alguém tenha
poderia disponibilizar no Dropbox???


Em 28 de maio de 2014 10:51, Hermann ilhadepaqu...@bol.com.br escreveu:

 http://pt.scribd.com/doc/38164469/Math-Olympiad-
 Problems-All-Countries-1989-2009

 Edited by:Amir Hossein Parvardi

 Esse cara compilou milhares de exercícios de olimpiadas colocando em um só
 PDF retirando do site

 www.artofproblemsolving.com/Forum/resources.php

 e tem mais coisas, feitas por ele http://www.math-olympiad.blogsky.com/

 o único problema do site do Amir é que está escrito em uma lingua  tipo
 árabe ou persa...mas muito em inglês.

 Como achei hiper interessante estou compartilhando o link com vocês espero
 que gostem.

 Não compilou nenhumna solução, mas está bem organizado, quem quiser com
 WINDOWS acrescentar ou retirar páginas entre em contato de email comigo
 para eu explicar um program que uso chamado pdftk.exe

 abraços
 Hermann

 ps: quem quiser compartilhar arquivos parecidos com esse, agradeço
 antecipadamente

 --
 Esta mensagem foi verificada pelo sistema de antivírus e
 acredita-se estar livre de perigo.

 =
 Instruções para entrar na lista, sair da lista e usar a lista em
 http://www.mat.puc-rio.br/~obmlistas/obm-l.html
 =


-- 
Esta mensagem foi verificada pelo sistema de antiv�rus e
 acredita-se estar livre de perigo.



[obm-l] Re: [obm-l] Re: [obm-l] Re: [obm-l] Re: [obm-l] compilação em um só PDF de milhares, muito mais de 5000

2014-05-28 Por tôpico Jeferson Almir
Pode sim!! Manda!!


Em 28 de maio de 2014 19:44, Arthur Max wm4x@gmail.com escreveu:

 Para  baixar do scribd faz assim, clica no link para baixar.. Aqui tá
 descarregar
 Depois clica em New to Scribd? Create a free account to get started.
 Coloca qualquer coisa nos campos de login, email e senha (você não irá
 precisar desses dados novamente).

 Bem no final da página tem bem assim..
 Active Scribd users who contribute quality content to the Scribd
 library are automatically given access to content otherwise only
 available to subscribers.
 Upload your original documents and download this document, or any and
 as many other documents as you'd like for free! Começar o Envio
 clique em Começar o Envio Coloque para fazer o upload.

 No seu computador crie qualquer arquivo .txt e preencha ele com
 qualquer coisa (Eu costumo colocar coisas do tipo blablabla e copiar
 e colar até ficar um texto bem grande), depois envie o arquivo pra lá,
 na descrição coloque qualquer coisa (costumo colocar a mesma coisa que
 tem no arquivo), na tag também coloque qualquer coisa (faço a mesma
 coisa que faço na descrição), escolha qualquer categoria, no preço
 deixe free mesmo.

 Na parte superior vai aparecer algo do tipo:
 Obrigado pela sua contribuição! Voltar para o documento para fazer
 download.

 Clique ai, ele vai redirecionar para a página do livro novamente, ao
 lado vai tá liberado o download.. Baixar e imprimir este documento,
 escolha o formato e baixe.

 Para baixar outra coisa, é só fazer o mesmo processo.

 Eu costumo fazer isso na aba anônima para o navegador não salvar
 nenhum dado, e depois precisar tá deslogando dessa conta que não será
 mais usada.

 Boa sorte com os downloads!

 []'s

 Em 28/05/14, Hermannilhadepaqu...@bol.com.br escreveu:
  Posso passar direto pro seu email? São 50 mega e ai vc coloca em um
 dropbox
  para os outros?
- Original Message -
From: Jeferson Almir
To: obm-l@mat.puc-rio.br
Sent: Wednesday, May 28, 2014 5:52 PM
Subject: [obm-l] Re: [obm-l] compilação em um só PDF de milhares, muito
  mais de 5000
 
 
Alguém conseguiu baixar no Scribd?? Não estou conseguindo caso alguém
  tenha poderia disponibilizar no Dropbox???
 
 
 
 
Em 28 de maio de 2014 10:51, Hermann ilhadepaqu...@bol.com.br
 escreveu:
 
 
 
 http://pt.scribd.com/doc/38164469/Math-Olympiad-Problems-All-Countries-1989-2009
 
  Edited by:Amir Hossein Parvardi
 
  Esse cara compilou milhares de exercícios de olimpiadas colocando em
 um
  só PDF retirando do site
 
  www.artofproblemsolving.com/Forum/resources.php
 
  e tem mais coisas, feitas por ele
 http://www.math-olympiad.blogsky.com/
 
  o único problema do site do Amir é que está escrito em uma lingua
  tipo
  árabe ou persa...mas muito em inglês.
 
  Como achei hiper interessante estou compartilhando o link com vocês
  espero que gostem.
 
  Não compilou nenhumna solução, mas está bem organizado, quem quiser
 com
  WINDOWS acrescentar ou retirar páginas entre em contato de email comigo
 para
  eu explicar um program que uso chamado pdftk.exe
 
  abraços
  Hermann
 
  ps: quem quiser compartilhar arquivos parecidos com esse, agradeço
  antecipadamente
 
  --
  Esta mensagem foi verificada pelo sistema de antivírus e
  acredita-se estar livre de perigo.
 
 
  =
  Instruções para entrar na lista, sair da lista e usar a lista em
  http://www.mat.puc-rio.br/~obmlistas/obm-l.html
 
  =
 
 
 
 
--
Esta mensagem foi verificada pelo sistema de antiv�rus e
acredita-se estar livre de perigo.
  --
  Esta mensagem foi verificada pelo sistema de antivírus e
   acredita-se estar livre de perigo.
 
 

 --
 Esta mensagem foi verificada pelo sistema de antivírus e
  acredita-se estar livre de perigo.


 =
 Instru�ões para entrar na lista, sair da lista e usar a lista em
 http://www.mat.puc-rio.br/~obmlistas/obm-l.html
 =


-- 
Esta mensagem foi verificada pelo sistema de antiv�rus e
 acredita-se estar livre de perigo.



[obm-l] Re: Equações Funcionais

2014-05-21 Por tôpico Jeferson Almir
O problema nao forneceu a forma implícita que deveríamos trabalhar ele
dizia apenas isso.. Mas eu acredito que seja isso mesmo que você
relacionou..

Em terça-feira, 20 de maio de 2014, terence thirteen 
peterdirich...@gmail.com escreveu:

 Ou seja, se a+c=2b então f(a)*f(b)=f(c)^2?


 Em 17 de maio de 2014 13:45, Jeferson Almir 
 jefersonram...@gmail.comjavascript:_e(%7B%7D,'cvml','jefersonram...@gmail.com');
  escreveu:

 Determine todas as funções contínuas que projeta três termos sucessivos
 de uma progressão aritmética em três termos de uma progressão geométrica.
 Desde já agradeço qualquer ajuda.
 --
 Esta mensagem foi verificada pelo sistema de antivírus e
 acredita-se estar livre de perigo.




 --
 /**/
 神が祝福

 Torres

 --
 Esta mensagem foi verificada pelo sistema de antivírus e
 acredita-se estar livre de perigo.

-- 
Esta mensagem foi verificada pelo sistema de antiv�rus e
 acredita-se estar livre de perigo.



[obm-l] Equações Funcionais

2014-05-17 Por tôpico Jeferson Almir
Determine todas as funções contínuas que projeta três termos sucessivos de
uma progressão aritmética em três termos de uma progressão geométrica.
Desde já agradeço qualquer ajuda.

-- 
Esta mensagem foi verificada pelo sistema de antiv�rus e
 acredita-se estar livre de perigo.



[obm-l] Re: [obm-l] Curiosidade sobre funções periódicas

2014-04-27 Por tôpico Jeferson Almir
Caro Artur eu soube agora :) como podemos provar isto???


Em 3 de março de 2013 01:51, Artur Costa Steiner
steinerar...@gmail.comescreveu:

 Esta é uma curiosidade mesmo. Faz lembrar um programa de rádio dos anos 60
 que começava assim Sabia você amigo ouvinte... E aí vinha algo muito
 interessante como o rei Louis XV gostava de laranja.

 Bom, sabia vc, amigo da lista, que, se o gráfico de f de R em R for
 simétrico com relação a 2 eixos verticais distintos, então f é periódica?
 Eu não sabia, descobri há alguns dias. Se os eixos forem x = a e x = b,
 então 2 |b - a| é um período.

 Abraços.

 Artur Costa Steiner
 --
 Esta mensagem foi verificada pelo sistema de antivírus e
  acredita-se estar livre de perigo.


 =
 Instru�ões para entrar na lista, sair da lista e usar a lista em
 http://www.mat.puc-rio.br/~obmlistas/obm-l.html
 =


-- 
Esta mensagem foi verificada pelo sistema de antiv�rus e
 acredita-se estar livre de perigo.



[obm-l] Re: Moedas vícios e jogos relacionados

2014-04-23 Por tôpico Jeferson Almir
E ai amigos sobre essas 2 questões e essas possíveis soluções há algum
furo??? Outra maneira mais trivial??? Abraço

Em terça-feira, 22 de abril de 2014, Jeferson Almir 
jefersonram...@gmail.com escreveu:

 Agora exponho no consenso que cheguei e que discuti com outros. ..

 PROBLEMA 1

 Passo 1. Jogue a moeda duas vezes e anote os 2 resultados.



 Passo 2.1) Se os resultados forem iguais ignore ambos e refaça o passo 1
 (não aproveite nenhum dos 2 resultados).



 Passo 2.2) Se os resultados forem diferentes, ignore o segundo resultado e
 fique com o primeiro.



 Veja que isso não serve para determinar se a moeda em si é justa ou não.
 Mas é um método onde você tem probabilidade 1/2 de obter cara e 1/2 de
 obter coroa, independentemente de qual a probabilidade de se obter cara em
 uma única jogada da moeda.



 PROBLEMA 2

  Probabilidade é sempre cheio de truques...acho que nesse caso a
 probailidade é a mesma (será?) Se a moeda viciada tiver probabilidade A de
 dar cara e B de dar coroa (onde A + B =1). Então a probabilidade de o
 lançamento da moeda viciada bater com o lançamento da moeda-sorteio é 1/2 A
 + 1/2 B = 1/2.

 Se a moeda-sorteio fosse também viciada (A,B) aí a probabilidade seria
 A^2 + B^2 que é sempre maior ou igual a 1/2.


 Em 22 de abril de 2014 19:17, Jeferson Almir 
 jefersonram...@gmail.comjavascript:_e(%7B%7D,'cvml','jefersonram...@gmail.com');
  escreveu:

 Caros certa vez discutimos tal temática e como aqui trata-se de uma lista
 de discussão retomo  com esses velhos 2 problemas:

 1. Um jogador pretende tomar uma decisão através do lançamento de uma
 moeda,  caso ocorra *coroa* ele viaja *cara* caso contrário,   porém ele
 sabe que ela é viciada então será possível ele tomar uma decisão justa sem
 que o vício da moeda interfira na sua decisão?? E mostre tal argumento.



 2. Uma competição de cara-ou-coroa; é lançada uma moeda normal, que é a
 moeda-sorteio, ganha o jogador cujo lance de moeda der o mesmo resultado.
 O jogador A joga outra moeda normal, que é sua aposta; o jogador B, no
 entanto, tem que fazer sua aposta a partir de uma moeda viciada, que tem
 maiores chances de cair em cara do que em coroa. Qual dos dois jogadores
 tem maiores chances de ganhar ao longo de uma série de rodadas?  Proposto
 certa vez por *Jorge Luís Rodrigues.*






-- 
Esta mensagem foi verificada pelo sistema de antiv�rus e
 acredita-se estar livre de perigo.



[obm-l] Moedas vícios e jogos relacionados

2014-04-23 Por tôpico Jeferson Almir
E ai amigos sobre essas 2 questões e essas possíveis soluções há algum
furo??? Ou uma maneira mais trivial??? Abraço

Em terça-feira, 22 de abril de 2014, Jeferson Almir 
jefersonram...@gmail.comjavascript:_e(%7B%7D,'cvml','jefersonram...@gmail.com');
escreveu:

 Agora exponho no consenso que cheguei e que discuti com outros. ..

 PROBLEMA 1

 Passo 1. Jogue a moeda duas vezes e anote os 2 resultados.



 Passo 2.1) Se os resultados forem iguais ignore ambos e refaça o passo 1
 (não aproveite nenhum dos 2 resultados).



 Passo 2.2) Se os resultados forem diferentes, ignore o segundo resultado e
 fique com o primeiro.



 Veja que isso não serve para determinar se a moeda em si é justa ou não.
 Mas é um método onde você tem probabilidade 1/2 de obter cara e 1/2 de
 obter coroa, independentemente de qual a probabilidade de se obter cara em
 uma única jogada da moeda.



 PROBLEMA 2

  Probabilidade é sempre cheio de truques...acho que nesse caso a
 probailidade é a mesma (será?) Se a moeda viciada tiver probabilidade A de
 dar cara e B de dar coroa (onde A + B =1). Então a probabilidade de o
 lançamento da moeda viciada bater com o lançamento da moeda-sorteio é 1/2 A
 + 1/2 B = 1/2.

 Se a moeda-sorteio fosse também viciada (A,B) aí a probabilidade seria
 A^2 + B^2 que é sempre maior ou igual a 1/2.


 Em 22 de abril de 2014 19:17, Jeferson Almir 
 jefersonram...@gmail.comescreveu:

 Caros certa vez discutimos tal temática e como aqui trata-se de uma lista
 de discussão retomo  com esses velhos 2 problemas:

 1. Um jogador pretende tomar uma decisão através do lançamento de uma
 moeda,  caso ocorra *coroa* ele viaja *cara* caso contrário,   porém ele
 sabe que ela é viciada então será possível ele tomar uma decisão justa sem
 que o vício da moeda interfira na sua decisão?? E mostre tal argumento.



 2. Uma competição de cara-ou-coroa; é lançada uma moeda normal, que é a
 moeda-sorteio, ganha o jogador cujo lance de moeda der o mesmo resultado.
 O jogador A joga outra moeda normal, que é sua aposta; o jogador B, no
 entanto, tem que fazer sua aposta a partir de uma moeda viciada, que tem
 maiores chances de cair em cara do que em coroa. Qual dos dois jogadores
 tem maiores chances de ganhar ao longo de uma série de rodadas?  Proposto
 certa vez por *Jorge Luís Rodrigues.*






-- 
Esta mensagem foi verificada pelo sistema de antiv�rus e
 acredita-se estar livre de perigo.



[obm-l] Moedas vícios e jogos relacionados

2014-04-22 Por tôpico Jeferson Almir
Caros certa vez discutimos tal temática e como aqui trata-se de uma lista
de discussão retomo  com esses velhos 2 problemas:

1. Um jogador pretende tomar uma decisão através do lançamento de uma moeda,
caso ocorra *coroa* ele viaja *cara* caso contrário,   porém ele sabe que
ela é viciada então será possível ele tomar uma decisão justa sem que o
vício da moeda interfira na sua decisão?? E mostre tal argumento.



2. Uma competição de cara-ou-coroa; é lançada uma moeda normal, que é a
moeda-sorteio, ganha o jogador cujo lance de moeda der o mesmo resultado.
O jogador A joga outra moeda normal, que é sua aposta; o jogador B, no
entanto, tem que fazer sua aposta a partir de uma moeda viciada, que tem
maiores chances de cair em cara do que em coroa. Qual dos dois jogadores
tem maiores chances de ganhar ao longo de uma série de rodadas?  Proposto
certa vez por *Jorge Luís Rodrigues.*

-- 
Esta mensagem foi verificada pelo sistema de antiv�rus e
 acredita-se estar livre de perigo.



[obm-l] Re: Moedas vícios e jogos relacionados

2014-04-22 Por tôpico Jeferson Almir
Agora exponho no consenso que cheguei e que discuti com outros. ..

PROBLEMA 1

Passo 1. Jogue a moeda duas vezes e anote os 2 resultados.



Passo 2.1) Se os resultados forem iguais ignore ambos e refaça o passo 1
(não aproveite nenhum dos 2 resultados).



Passo 2.2) Se os resultados forem diferentes, ignore o segundo resultado e
fique com o primeiro.



Veja que isso não serve para determinar se a moeda em si é justa ou não.
Mas é um método onde você tem probabilidade 1/2 de obter cara e 1/2 de
obter coroa, independentemente de qual a probabilidade de se obter cara em
uma única jogada da moeda.



PROBLEMA 2

 Probabilidade é sempre cheio de truques...acho que nesse caso a
probailidade é a mesma (será?) Se a moeda viciada tiver probabilidade A de
dar cara e B de dar coroa (onde A + B =1). Então a probabilidade de o
lançamento da moeda viciada bater com o lançamento da moeda-sorteio é 1/2 A
+ 1/2 B = 1/2.

Se a moeda-sorteio fosse também viciada (A,B) aí a probabilidade seria
A^2 + B^2 que é sempre maior ou igual a 1/2.


Em 22 de abril de 2014 19:17, Jeferson Almir jefersonram...@gmail.comescreveu:

 Caros certa vez discutimos tal temática e como aqui trata-se de uma lista
 de discussão retomo  com esses velhos 2 problemas:

 1. Um jogador pretende tomar uma decisão através do lançamento de uma
 moeda,  caso ocorra *coroa* ele viaja *cara* caso contrário,   porém ele
 sabe que ela é viciada então será possível ele tomar uma decisão justa sem
 que o vício da moeda interfira na sua decisão?? E mostre tal argumento.



 2. Uma competição de cara-ou-coroa; é lançada uma moeda normal, que é a
 moeda-sorteio, ganha o jogador cujo lance de moeda der o mesmo resultado.
 O jogador A joga outra moeda normal, que é sua aposta; o jogador B, no
 entanto, tem que fazer sua aposta a partir de uma moeda viciada, que tem
 maiores chances de cair em cara do que em coroa. Qual dos dois jogadores
 tem maiores chances de ganhar ao longo de uma série de rodadas?  Proposto
 certa vez por *Jorge Luís Rodrigues.*





-- 
Esta mensagem foi verificada pelo sistema de antiv�rus e
 acredita-se estar livre de perigo.



Re: [obm-l] JEITO CEARENSE!

2014-04-20 Por tôpico Jeferson Almir
Probabilidade é sempre cheio de truques...acho que nesse caso a
probailidade é a mesma (será?) Se a moeda viciada tiver probabilidade A de
dar cara e B de dar coroa (onde A + B =1). Então a probabilidade de o
lançamento da moeda viciada bater com o lançamento da moeda-sorteio é 1/2 A
+ 1/2 B = 1/2.

Se a moeda-sorteio fosse também viciada (A,B) aí a probabilidade seria
A^2 + B^2 que é sempre maior ou igual a 1/2.

Em quarta-feira, 12 de setembro de 2012, Jorge Luis Rodrigues e Silva Luis 
jorgelrs1...@hotmail.com escreveu:

  Tem razão, Jeferson! Pois eu até duvido que o raciocinio do Tarsis seja
 equivalente ao seue jogando mais lenha na fogueira...

 Uma competição de cara-ou-coroa; é lançada uma moeda normal, que é a
 moeda-sorteio, ganha o jogador cujo lance de moeda der o mesmo resultado.
 O jogador A joga outra moeda normal, que é sua aposta; o jogador B, no
 entanto, tem que fazer sua aposta a partir de uma moeda viciada, que tem
 maiores chances de cair em cara do que em coroa. Qual dos dois jogadores
 tem maiores chances de ganhar ao longo de uma série de rodadas? Por alguma
 razão tenho a sensação de que o apostador com a moeda viciada tem mais
 chances, mas não sei?

 Tenho a impressão de que se eu deixar uma moeda parada em cara na mesa,
 e ficar jogando uma outra N vezes, elas tem mais chances de estarem ambas
 em cara ao longo de várias jogadas, do que teriam duas moedas jogadas de
 cairem do mesmo lado. Dá impressão que o primeiro caso tem 50% de chance
 normal do cara-ou-coroa, enquanto que na segunda situação as
 improbabilidades se somam ou se multiplicam de alguma forma ou sei lá.?

 --
 Date: Tue, 11 Sep 2012 22:59:53 -0300
 Subject: Re: [obm-l] JEITO CEARENSE!
 From: 
 jefersonram...@gmail.comjavascript:_e(%7B%7D,'cvml','jefersonram...@gmail.com');
 To: obm-l@mat.puc-rio.brjavascript:_e(%7B%7D,'cvml','obm-l@mat.puc-rio.br');

 Jorge Luis Vc se refere a este Problema pois não tive ainda um consenso
 sobre a mesma??
 Dada uma Moeda viciada e uma pessoa  deseja fazer uma escolha utilizando
 tal moeda,(por exemplo se caso ela nao fosse viciada ele atribuiria cara
 para sim e coroa para nao). Como ele deve proceder para realizar tal
 escolha com a moeda de maneira a realizar sua escolha de maneira que o
 vicio da moeda nao interfira???

 Em 9 de setembro de 2012 15:24, Willy George Amaral Petrenko 
 wgapetre...@gmail.comjavascript:_e(%7B%7D,'cvml','wgapetre...@gmail.com');
  escreveu:

 Na verdade p^2+(1-p)^2 =1/2 das possibilidades são descartadas.


 2012/9/9 Jorge Luis Rodrigues e Silva Luis 
 jorgelrs1...@hotmail.comjavascript:_e(%7B%7D,'cvml','jorgelrs1...@hotmail.com');
 

  Olá Pessoal!  Ainda com relação ao vício da moeda do Jeferson observem
 que metade das possibilidades são descartadas...Como bolar uma variante ao
 esquema tal que minimizasse essa perda? Será que o Neumann acharia uma
 saída?

 Numa faculdade há dois cursos e um rapaz e uma moça estão trocando idéias.
 O rapaz diz: Aqui eles discriminam contra os homens, a proporção de homens
 admitidos (dentre os candidatos) é menor do que a de mulheres. A moça
 responde: Não, eles discriminam contra as mulheres. Nos dois cursos a
 proporçào de mulheres admitidas (dentre as candidatas) é menor do que a de
 homens. É possivel que ambos tenham razào quanto aos fatos?  (Proposto
 pelo Nicolau!)


 A propósito! De quantas formas podemos colocar N rainhas em um tabuleiro N
 * N tal que nenhuma rainha possa enxergar outra?


 Abraços!





-- 
Esta mensagem foi verificada pelo sistema de antiv�rus e
 acredita-se estar livre de perigo.



Re: [obm-l] off topic - livro caronnet

2014-04-14 Por tôpico Jeferson Almir
Idem. .


Em 14 de abril de 2014 21:05, marcone augusto araújo borges 
marconeborge...@hotmail.com escreveu:

 Eu quero.

 --
 Date: Mon, 14 Apr 2014 13:35:45 -0700
 From: regisgbar...@yahoo.com.br
 Subject: Re: [obm-l] off topic - livro caronnet
 To: obm-l@mat.puc-rio.br

 Olá Pessoal
 No ano passado teve contato com todos os livros do caronnet e fiz um scan
 deles caso vocês queiram é só pedir que mando link para o email pessoal.

 Regis
   Em Segunda-feira, 14 de Abril de 2014 14:34, Sergio Lima 
 sergi...@smt.ufrj.br escreveu:
  oi Hermann,

 Procure na Estante Virtual. Acho que voce encontrarah lah.
 Os livros tem excelentes exercicios.

 Abraco,
 sergio


 2014-04-20 13:06 GMT-03:00 Hermann ilhadepaqu...@bol.com.br:

  OFF - TOPIC

 Meus amigos, gostaria de saber se alguém já viu os livros do caronnet
 volumes 1, 2, 4 e 5.

 Porque há anos eu procuro e nunca vi.

 E a coleção é boa, na opinião de vocês?

 abraços
 Hermann

 --
 Esta mensagem foi verificada pelo sistema de antivírus e
 acredita-se estar livre de perigo.



 --
 Esta mensagem foi verificada pelo sistema de antivírus e
 acredita-se estar livre de perigo.



 --
 Esta mensagem foi verificada pelo sistema de antivírus e
 acredita-se estar livre de perigo.

 --
 Esta mensagem foi verificada pelo sistema de antivírus e
 acredita-se estar livre de perigo.


-- 
Esta mensagem foi verificada pelo sistema de antiv�rus e
 acredita-se estar livre de perigo.



Re: [obm-l] infinitas ternas

2014-03-05 Por tôpico Jeferson Almir
Eu fiz b=5b' ai eu simplifiquei os fatores comum.. Logo depois fiz c=15c' e
simplifiquei as fatores comuns e tenho b'^2 -15c'^2=1


Em segunda-feira, 3 de março de 2014, marcone augusto araújo borges 
marconeborge...@hotmail.com escreveu:

 Já mandei duas mensagens e nada.Eu não entendi como
 o Jeferson chegou em b^2 - 15c^2 = 1


 --
 Esta mensagem foi verificada pelo sistema de antivírus e
 acredita-se estar livre de perigo.


-- 
Esta mensagem foi verificada pelo sistema de antivírus e
 acredita-se estar livre de perigo.



Re: [obm-l] infinitas ternas

2014-02-27 Por tôpico Jeferson Almir
Fazendo a=31 obtemos 3bˆ2-5cˆ2=75  = 3 e 5 são multiplos de 75  =
existem b=5b' e c=15c' tais que. ... .. . = b'ˆ2 -15c'ˆ2=1 (equação de
Pell ) onde a partir de uma solução particular podemos (b_0, c_0) podemos
gerar infinitas então (b',c')=(4,1) = e todas serão da forma (31,5b',
15c'). Peço desculpas pela má organização.


Em 27 de fevereiro de 2014 19:21, marcone augusto araújo borges 
marconeborge...@hotmail.com escreveu:

 Sejam a,b e c números inteiros positivos.Mostre que existem infinitas
 ternas (a,b,c)  que são soluções da equação 2a^2 + 3b^2 - 5c^2 = 1997

 --
 Esta mensagem foi verificada pelo sistema de antivírus e
 acredita-se estar livre de perigo.


-- 
Esta mensagem foi verificada pelo sistema de antiv�rus e
 acredita-se estar livre de perigo.



Re: [obm-l] [off topic] Apostila Desenho Geometrico Prof Brandao

2014-02-20 Por tôpico Jeferson Almir
Eu reforço a indagação do Sergio pois eu ainda tenho interesse nesse
material pois participo do programa  POT financiado pelo governo aqui em
fortaleza na parte de geometria e necessito trabalhar  essa necessidade que
os meninos possuem em desenho e acredito que esse material seria útil.
Cordialmente Jeferson Almir

Em quinta-feira, 20 de fevereiro de 2014, Mauricio de Araujo 
mauricio.de.ara...@gmail.com escreveu:

 Sérgio,

 As apostilas eram do curso Impacto do Rio, já há tempos falido... não me
 recordo de ter visto nada na apostila original mencionando copyright... vou
 verificar de novo... mas para ter certeza de que não estamos infringindo
 nenhuma lei, o mais correto é fazer uma consulta a um advogado. Pelo menos
 não foi o caso de auferirmos lucro com a disponibilização do material..

 Entendo que o que não está proibido é permitido; se não houver nada
 proibindo a divulgação do material então não estamos cometendo crime. Mas
 realmente precisamos checar...


 2014-02-20 15:47 GMT-03:00 Sergio Lima 
 sergi...@smt.ufrj.brjavascript:_e(%7B%7D,'cvml','sergi...@smt.ufrj.br');
 :

 Caros Colegas,

 Desculpem-me por retomar esse tema e faze-lo de forma ainda mais
 [off topic] que antes.

 Alguem poderia me dizer se a divulgacao desse material iria
 infringir alguma regra/lei/recomendacao relativa aos direitos autorais?

 Sei que isso estah longe de ser o tema desta lista,
 mas como isso tambem foi o assunto de tantas mensagens,
 eu pensei que alguem da lista saberia me esclarecer isto.

 Agradeco antecipadamente por qualquer informacao.

 Abraco,
 sergio


 --
 Esta mensagem foi verificada pelo sistema de antivírus e
 acredita-se estar livre de perigo.




 --
 Abraços

 oɾnɐɹɐ ǝp oıɔıɹnɐɯ
 *momentos excepcionais pedem ações excepcionais.*
 *Os cemitérios estão cheios de pessoas insubstituíveis em seus ofícios.*

 --
 Esta mensagem foi verificada pelo sistema de antivírus e
 acredita-se estar livre de perigo.

-- 
Esta mensagem foi verificada pelo sistema de antiv�rus e
 acredita-se estar livre de perigo.



  1   2   >